SlideShare a Scribd company logo
1 of 59
ชุดที่ 1 Medicine
                   1.) ตอบ 4. Defibillation
                        Ventricular fibrillation - EKG : no p wave ,no QRS cpx can be identified, irregular
                   rhythm ,rate >300




                                                               RA J
                        Management : VF and pulseless : Defibillation
                              เพิ่มเติม VF และคลํา pulseได



                                            RI
                              1.vital sign stable : amiodarone



                                         SI
                              2.vital sign unstable(shock) : cardioversion
                   2.) ตอบ 1. 3rd heart block



                     AJ
                        3rd heart block พบวา P กับ QRS จะไมสัมพันธกัน และจะมีอาการไดบอย, 2nd




                  IR
                   mobitz I พบวา PR ยาวขึ้นเรื่อยๆ จน QRS หายไปเปนบางครั้งแลวจึงกลับมาเปนปกติ ไม




                IR
                   คอยมีอาการ, 2nd mobitz II พบวา PR ยาวเทาๆเดิม แตอยูๆ QRS ก็หายไป พบอาการได
                   บาง, 1st degree พบ PR ยาวขึ้นเฉยๆ แลวก็ไมมีอาการ


              S    3.) ตอบ 4. adrenaline im



         AJ
                        Anaphylaxis criteria dx มี 3วิธี




      IR
                        1.มีอาการทางskin + Rs dysfn or BP drop




 IR
                        2.มี Hx สัมผัสสารที่นาจะเปน allergen + อาการ 2/4 (1.skin or mucosa 2.RS dysfn 3.
                   BP drop 4.GI)


S                       3.มี Hx สัมผัส known allergen + SBP < 90 หรือ ลดลงจาก30% baseline
                        Management : adrenaline im 1:1000 dose 0.01 ml/kg
                           ( child max 0.3ml, adult 0.5ml) ใหซ้ําได 1-2 ครั้ง หางกัน 10-15 นาที
                   4.) ตอบ 5. Paroxysmal nocturnal hemoglobinuria(PNH)
                        PBS : NCNC / เปน intravascular hemolysis (ปสสาวะมีสีชาหรือโคลา) , อาการ
                   แสดง : pancytopenia
การวินิจฉัยจะตองทํา Ham’s test (acid serum test) หรือ sucrose hemolysis test             7.) ตอบ 2. Prostigmine test
     choice ขออื่น AIHA , HS ,Thalassemia เปน extravascular hemolysis                        เนื่องจากนึกถึง Myasthenia gravis / Hx&PE : fluctuating weakness increased by
     G-6-PD เปน intravascular hemolysis / PBS : bite cell ,Heinz bodies                  exertion, Weakness increases during the day and improves with rest, proximal weakness
มักเปนผูชาย(XR) และมักมีประวัติ drug-induced                                            greater than distal muscle weakness, ptosis, limited to the EOM



                                                                                                                                           J
5.) ตอบ 3. AIHA                                                                                Ix : NEOSTIGMINE TEST ( Prostigmin test) 1-2 mg im evaluate 20-40 mins



                                                                                                                                        RA
     จาก ญ วัยรุน ซีด เหลืองเล็กนอย ตับไมโต มามโตเล็กนอย reti ขึ้น (แตทําไมMVC           Rx : Anticholinesterase medication (oral pyridostigmine) เปน symptomatic



                                                                                                                    RI
เล็ก ซึ่งจะนึกถึง thalassemia, iron def. หากจะตอบ thalassemia ตองเปน H disease ที่      treatment; onset 15-30 mins, last for 3-4 hrs; begin with moderate dose 60mg 3-5 times




                                                                                                                 SI
สามารถมี reti ขึ้นไดประมาณ 5 %)                                                          per day, maximal dose 120mg every 3 hrs during daytime; adverse effect : increased
     Thalassemia ไมมีอาการ จนถึง ซีดเหลืองรุนแรง ตับมามโต chipmuck face                 muscle weakness and muscarinic effect, antidote is propantheline bromide




                                                                                  AJ
hyperpigmentation MCV เล็ก                                                                8.) ตอบ 1. ขอความยินยอมพอแมเพื่อ LP




                                                                               IR
     Iron def ซีด เหนื่อย มีประวัติเสียเลือดเรื้อรัง glossitis, koilonychias, angular          ผูปวยเด็กอายุ 8 เดือน มีอาการไขสูง มี Brudzinski’s sign positive จากโจทยทําใหนึก
stomatitis,Wbc,plt ปกติ พบ serum ferritin, Serum Iron, Transferrin Saturation ต่ํา TIBC   ถึงภาวะ meningitis มากที่สุดผูปวยรายนี้ อาการที่พบไดบอยของภาวะนี้คือ ไข ปวดศีรษะ



                                                                             IR
สูง MVC เล็ก                                                                              ซึม สับสน คลื่นไสอาเจียน ปวดคอ คอแข็ง ตรวจรางกายอาจพบ ระดับความรูสึกตัวที่



                                                                           S
     G6PD มักพบใน ช> ญ ไมพบมามโต ซีดฉับพลันและเปนชั่วคราว อาจเหลือง                    ลดลง และอาการแสดงของ Meningeal irritation ทั้ง Kernig’s และ Brudzinski’s sign อาจ




                                                  AJ
เล็กนอย อาจมีปสสาวะดํา พบ Heinz body, ghost cell                                        มี nuchal rigidity นอกจากนี้ยังอาจพบความผิดปกติของ CN รวมดวย เชน III ,IV, VIII /
     HS มีอาการรุนแรงเมื่อมีภาวะติดเชื้อหรือมีไข มาพบแพทยดวย เหลือง มักพบมามโต



                                               IR
                                                                                          การวินิจฉัย gold standard ในผูปวยที่สงสัยภาวะนี้ตองทํา Lumbar puncture เพื่อดู CSF
มักพบภาวะแทรกซอนเปนนิ่วในถุงน้ําดี พบ spherocyte, reticulocyte มากขึ้น                  profile วาเปนmeningitis หรือไม และเกิดจากเชื้อกลุมใด อยางไรก็ตาม มีขอหามในการทํา



                               IR
     AIHA warm type – ญ>ช อายุ 20-40 ตรวจพบโลหิตจาง เหลืองเล็กนอย อาจจบมาม              LP ถาผูปวยมี Focal neurologic sign,มี Evidence of space-occupying lesion with



                              S
หรือตับโตเล็กนอย reticulocyte มากขึ้น wbc ต่ํา หรือ สูงขึ้น plt ปกติ                     increased ICP , Immunocompromised host, , มี new onset of seizure ภายใน 1 สัปดาห
6.) ตอบ 5. FFP                                                                            และมี papilledema ผูที่มีขอหามขอใดขอหนึ่ง ควรไดรับการทํา CT brain กอน เนื่องจาก
     I/C of FFP - 1.replace of multiple coagulation factors ex. liver disease , DIC ,     กลัว brain herniation ถาพบรอยโรคใน brain จริง จะไมทํา LP แลว treat ไปเลย
massive transfusion(Pt,APTT ratio>1.5x normal)                                                 การรักษาคือ การให Empirical antibiotics ทันทีหลังจากการทํา LP และ Hemoculture
     2.Treatment of TTP, HUS                                                              แลว นอกจากนี้การให dexamethasone ยังมีประโยชนในผูปวยที่เปน pneumococcal
                                                                                          meningitis
กลับมาพูดถึงผูปวยรายนี้ มีปญหาคือมี anterior fontanelle โปง เขาใจวานาจะหมายถึง   including sleepiness and impulse control disorders, these side effects resolve upon
มี ICP สูง เราไมแนใจวาเทาที่โจทยใหมาเปน C/I ในการทํา LP หรือไม แตคิดวาไมนาจะ     lowering the dose or discontinuing the medication
หามทํา LP เพราะเด็กยังไมมีอาการ focal neurologic sign ที่ชัดเจน (มั้ง?) ดังนั้นขอตอบขอ          - For patients younger than 65 years, use a dopamine agonist and then add
1 คือขอความยินยอมจากพอแมเพื่อ LP (แตไมแนใจวาตอง CT กอนรึปาวนะ แตหลักการ            levodopa/PDI when the dopamine agonist (with or without an MAO-B inhibitor) no



                                                                                                                                             J
ตามนี้ละ)                                                                                   longer provides good control of motor symptoms. Dopamine agonists may provide good



                                                                                                                                          RA
9.) ตอบ 5. ให amitriptyline                                                                 symptom control for several years.




                                                                                                                       RI
     อาการปวดศีรษะแบบตุบๆขางเดียว เปนๆหายๆ คิดถึง Migraine without aura ซึ่ง                    - For patients who are demented or older than 70 years (those who may be prone to




                                                                                                                    SI
หลักการรักษาคือ 1. หลีกเลี่ยงสิ่งกระตุน & 2. Medication โดย                                 adverse effects, such as hallucinations, from dopamine agonists), and for those likely to
     Acute : paracetamol, NSAIDs; Ergotamine tartrate; Triptans ( 5HT agonists)              require treatment for only a few years, not to use a dopamine agonist and depend on




                                                                                    AJ
     Chronic : ใหยา : β blocker, CCB, TCA, SSRIs เพื่อปองกันเมื่อ                          levodopa/PDI as primary symptomatic therapy




                                                                                 IR
           1. ปวดบอยมากกวา 2 ครั้งตอเดือน ซึ่งทําใหเกิด disability มากกวา 3 วัน               - benhexal(Anticholinergic) medications provide good tremor relief in
           2. ปวดแตละครั้งรุนแรงมาก                                                         approximately 50% of patients but do not improve bradykinesia or rigidity



                                                                               IR
           3. ปวดมากกวา 2 ครั้งตอสัปดาห                                                        - Selegiline (MAO-B inhibitors) provide mild symptomatic benefit, have excellent



                                                                             S
           4. ไมสามารถใชยา acute medication ได                                            side effect profiles, and may improve long-term outcomes, a good choice as initial




                                                   AJ
           5. ใชยา acute medication มากเกินไป                                               treatment for many patients, delays the need for levodopa therapy in early Parkinson
10.) ตอบ 2. levodopa



                                                IR
                                                                                             disease
     Parkinson disease is a progressive neurodegenerative disorder associated with a         11.) ตอบ 1. Phenytoin



                                IR
loss of dopaminergic neurons in the substantia nigra / Cardinal signs : resting tremor,           คิดถึงภาวะ generalized tonic-clonic seizures ยาหลักในการรักษาก็คือ Phenytoin



                               S
cogwheel rigidity, bradykinesia, Postural instability                                        Phenobarbital Carbamazepine สวน diazepam นั้นมักเลือกใชเปนยาชนิดแรกในกรณี
     - levodopa: standard of symptomatic treatment for Parkinson disease, fewest             status epilepticus
adverse effects in the short term, its chronic use is associated with the development of     12.) ตอบ 1. Hypokalemia
fluctuations and dyskinesias.                                                                     แขนขาออนแรง ตองคิดถึง hypokalemia ซึ่งรายนี้มีประวัติได HCTZ ซึ่งทําให
     - Bromocriptine(Dopamine agonists): provide moderate symptomatic benefit and            hypokalemia ได
rarely cause fluctuations and dyskinesias , but more side effects than levodopa,
13.) ตอบ 4. Ceftriaxone                                                                    ชวงอายุประมาณ 30-40 yrs อาจพบ IgA nepropathy, postinfectious, lupus ..... / การรักษา
      เนื่องจากผูปวยมีภาวะ urosepsis กลาวคือมีอาการไขสูง หนาวสั่น ปสสาวะแสบขัด        ใหเปน ACEI or ARB , steroid
ผล U/A พบ WBC 50-60 จึงเขาไดกับภาวะ UTI รวมกับมี sign of SIRS > 2ใน 4 ขอ ไดแก        17.) ตอบ 5. Primary polydipsia
1. T > 38 or < 36 C 2. RR > 24 3. P > 90 4. WBC > 12000 or < 4000 or band form > 10             ขอนี้บอกตามตรงวา ขอสอบมันงงๆนะ คือถาโจทยจําตัวเลขมาถูกแลว ก็ตองมาแปล



                                                                                                                                          J
% จึงควรใหการรักษาดวย IV quinolone / ampicillin + gentamicin / ceftriaxone               ผล lab กันนะ คนนี้มี hyponatremia แลว urine Na ต่ํา (ปกติ 15-250 mEq/L/day) และ



                                                                                                                                       RA
14.) ตอบ 1. urineVMA                                                                       urine osmol. ต่ํา (ปกติ 500-800 mOsm/kg) มาตัด choice กัน




                                                                                                                    RI
      Pheochromocytoma                                                                          SIADH คือ รางกายหลั่ง ADH ซึ่งทําหนาที่ดูดกลับน้ํา มากเกินไป ก็ทําให




                                                                                                                 SI
      อาการและอาการแสดง 1. Headache 2.Diaphoresis 3.Palpitations 4.Severe                  hyponatremia ได แต urine ก็จะตอง เขมขนสิ ควรมี urine osmol และ urine Na สูง จึงไม
hypertension (These 4 characteristics together are strongly suggestive of a                ตอบขอนี้




                                                                                  AJ
pheochromocytoma) 5.Tremor 6.Nausea 7.Weakness 8.Anxiety 9.Epigastric pain                      Low salt intake กินเกลือนอยไมใชสาเหตุที่ทําให hyponatremia ไดมากขนาดนี้




                                                                               IR
10.Flank pain 11.Constipation 12.weight loss                                                    Renal wasting คือ เสีย Na ไปทางไต ก็ควรมี urine Na สูงสินะ
      Dx 1.Plasma metanephrine testing has the highest sensitivity (96%) for detecting a        Thiazide มี side effect ทําให hyponatremia ไดจากการที่ขับ Na ทางปสสาวะเพิ่มขึ้น



                                                                             IR
pheochromocytoma, lower specificity (85%).                                                 ก็นาจะมี urine Na สูงดวย



                                                                           S
            2.24-hour urinary collection for catecholamines ,vanillylmandelic acid and          Primary polydipsia เขาไดมากสุด เพราะทําใหมี hyponatremia , urine Na ต่ํา และ




                                                  AJ
metanephrines has a sensitivity of 87.5% and a specificity of 99.7%                        urine osmol. ต่ํา ; แตก็ไมเขาใจวาเกี่ยวอะไรกับประวัติ CA lung ตอนแรกคิดวาจะให
            3.CT , MRI abdomen                                                             คิดถึง SIADH ที่พบไดใน Small cell lung cancer แต lab ไมเขาอะ



                                               IR
15.) ตอบ 3. Serum electrolyte                                                              18.) ตอบ 1. Calcium gluconate



                               IR
      ในผูปวยรายนี้จากประวัติและการตรวจรางกายทําใหนึกถึงภาวะ primary                        เนื่องจากมีภาวะ hyperkalemia (K =6.0)ทําใหเกิด cardiac arrhythmia ได ; อาการ



                              S
hyperaldosteronism เนื่องจากมี muscle weakness จาก potassium ต่ํา ไมมีอาการใจสั่น         ไดแก Generalized fatigue,Weakness ,Paresthesias ,Paralysis ,Palpitations หรืออาจ
เหงื่อ ออกหนาแดง อันจะพบไดใน pheochromocytoma ดังนั้นการสง Ix ที่เหมาะสมที่สุด          asymptomatic / Risk ไดแก Acute or chronic renal failure, Trauma including crush
ในผูปวยรายนี้ นาจะเปน serum electrolyte มากที่สุด                                      injuries (rhabdomyolysis) or burns ,Ingestion of foods high in potassium (eg, bananas,
16.) ตอบ 4. Prednisolone                                                                   oranges, high-protein diets, tomatoes, salt substitutes) ,ยา - Potassium supplements,
      ผูปวยมี nephrotic nephritic symdrome คิดถึง glomerular disease : primary or        potassium-sparing diuretics, NSAIDs, beta-blockers, digoxin, succinylcholine, and
secondary จากโจทยไมไดบอก clue ที่สงสัยวามีโรคประจําตัวใดๆ / คิดถึง primary กอน        digitalis glycoside ,Redistribution - Metabolic acidosis (diabetic ketoacidosis [DKA]),
catabolic state Rx : สงตรวจEKG,หยุดpotassium sparing drug, ให Calcium chloride or      HRCT ; CXR : Cystic shadow, bronchial wall thickening (Tramline and ring shadows);
calcium gluconate (Kalcinate)                                                            Treatment : postural drainage, ATB, Bronchodilator (ถามี asthma, COPD รวมดวย)
Adult                                                                                    21.) ตอบ 1. Sputum AFB
- Calcium chloride: 5 mL of 10% sol IV over 2 min (stop infusion if bradycardia )             เนื่องจาก อาการที่เขาไดกับ pulmonary TB คือ ไขตอนเย็นๆ ไอมีเสมหะ 3 wk ตรวจ



                                                                                                                                         J
- Calcium gluconate: 10 mL of 10% sol IV over 2 min (stop infusion if bradycardia )      film พบ alveolar infiltration และ Cavity ที่ RUL (suggest actibe TB) การตรวจพบ AFB



                                                                                                                                      RA
Pediatric                                                                                positive ก็จะชวยยืนยันผลได (แตถาตรวจไมพบก็คง treat as TB อยูดี?) Regimen ที่ควร




                                                                                                                   RI
- Calcium chloride: 0.2 mL/kg/dose of 10% sol IV over 5 min; not to exceed 5 mL (stop    ไดรับในการรักษาคือ 2 IRZE + 4 IR [Co-amoxiclav และ levofloxacin จริงๆแลวก็




                                                                                                                SI
infusion if bradycardia )                                                                สามารถใชในการรักษา TB ไดเชนกัน โดยใชรักษาในกลุม multidrug resistance TB ซึ่ง
- Calcium gluconate: 100 mg/kg (1 mL/kg) of 10% sol IV over 3-5 min; not to exceed 10    ตองใหรวมกับยาอื่นๆอีก คือ Clofazimene Moxifloxacin และ Amikacin]




                                                                                AJ
mL (stop infusion if bradycardia )                                                            ในผูปวย HIV จะมีโอกาสตรวจพบเชื้อจากเสมหะไดนอย ดังนั้นในผูปวย HIV เพื่อ




                                                                             IR
19.) ตอบ 1. Hypo K                                                                       การวินิจฉัยอาจตรวจโดย bronchoscopy, tissue biopsy, blood culture
     ก็ขอนี้ ถาม S-E ของ HCTZนะ ไดแก                                                 22.) ตอบ 2. วินิจฉัยใหไดวาเปนมะเร็งจริง



                                                                           IR
     Hypo “ขนม” K Na Mg แลวก                                                                แตสําหรับโจทยขอนี้ คิดวา เนื่องจากแคสงสัยวาเปนมะเร็งปอดกอนอื่นก็ตองวินิจฉัย



                                                                         S
     Hyper Ca glycemia lipidemia uricemia                                                ใหไดวาเปนมะเร็งปอดจริง จึงคิดวานาจะตอบขอ 2 นะ ขางลางนี้เปนความรูที่ไปหามาให




                                                 AJ
     สําหรับอาการแขนขาออนแรง เนี่ยกcommon จาก Hypo K แหละ                              ไมเกี่ยวกับการตอบขอนี้เลย
     Hypo Na N/V fatique ซึม ชัก                                                              โรคมะเร็งปอด : มีสาเหตุจากสารเคมีอนุภาคหลายชนิดไดแก แอสเบสตอส เรดอน



                                              IR
     Hypo Ca Neuromuscular irritability                                                  คลอโรเมธินอีเธอร โพลีอะโรมาติก ไฮโดรคารบอน อารเซนิก โครเมียม นิกเกิล อยางไรก็



                               IR
     HypoMg N/V CNS suppression hyporeflexia (คลายๆ preeclampsiaที่ตอง ใหMg           ตามจากประวัติการสัมผัสทําใหสามารถวินิจฉัยวาเกิดจากการทํางาน ประวัติที่ทําให



                              S
แกเรยเนอะ ^^)                                                                           วินิจฉัยยากคือ ประวัติการสูบบุหรี่ สวนใหญเมื่อวินิจฉัยกอนจะโตมากแลว และผูปวยจะ
20.) ตอบ 2. Bronchiectasis                                                               ถึงแกกรรมในเวลาไมนาน อาการคือมีอาการของการระคายเคืองของเยื่อหุมปอด
     เนื่องจากไอเรื้อรังมานาน มีเสมหะเขียว และ peribronchial thickening **               หลอดลม ถุงลมในปอด ทําใหมีอาการหอบเหนื่อย ไอเปนเลือด นอกจากนี้มะเร็งปอดยัง
Bronchiectasis เปน chronic infection ทําใหbronchus+bronchiole dilate อยางถาวร (เปน   สามารถแพรกระจายไปยังอวัยวะอื่นทําใหมีอาการของอวัยวะนั้นๆ การสูบบุหรี่ทําใหเปน
Obstructive lung) , เกิดจากเชื้อ : H. influenzae, S. pneumo, S. aureus, P. aeruginosa;   มะเร็งปอดไดและมีโอกาสที่จะเปนจากบุหรี่มากกวาการประกอบอาชีพหลายเทา
อาการ : ไอเรื้อรัง, เสมหะเขียวปริมาณมาก (อาจมีเลือดปน); Ix: CXR, Sputum culture,
23.) ตอบ 4. Bronchogenic carcinoma                                                           Complicated pleural effusion มีลักษณะดังนี้
     อาการแนนหนาอก เหนื่อย หนาและแขนบวม มี face and upper extremity edema ,                  1.Frank pus or empyema thoracis
superficial vein dilatation at chest wall เขาไดกับ SVC (ในระยะแรกอาจจะ ไมแสดง                2.มีorganisim (พบในผูปวยรายนี้)
อาการใด ๆ โดยเฉพาะเปน partial obstruction แตโดยสวนใหญจะมีอาการ และอาการ                     3.pH<7.2



                                                                                                                                              J
แสดงเล็ก ๆ นอย ๆ ใหเห็นบาง แตหาก total obstruction จะแสดงอาการตางๆโดย                      4.Sugar<60mg/dl (พบในผูปวยรายนี้)



                                                                                                                                           RA
Dyspnea เปนอาการที่พบมากที่สุด อาการอื่นๆ ประกอบดวย facial swelling, head                     5.Loculate parapneumonic effusion




                                                                                                                          RI
fullness, cough, arm swelling, chest pain, dysphagia, orthopnea, distorted vision,                                            Transudate    Exudate       Empyema



                                                                                                                        I
hoarseness, stridor, headache, nasal stuffiness, nausea, pleural effusions, and light-                                                                     thoracis



                                                                                                          S
headedness).                                                                                 pH                                7.4-7.6         <7.4          <7.2



                                                                                                        J
สาเหตุของ SVC obstruction                                                                    Cell                              <1,000      1,000-5,000   >5,000 PMN



                                                                                            RA
     มากกวา 80% เกิดจาก malignant mediastinal tumors : Bronchogenic carcinomas                                                 L,Mo           PMN




                                                                                         RI
พบ 75-80% ในภาวะนี้ โดยสวนใหญเปน small-cell carcinomas ; Non-Hodgkin                      Specific gravity                  <0.015         >0.015




                                                       SI
lymphoma (โดยเฉพาะ large cell type) พบประมาณ 10-15% ; Malignant อื่นๆที่พบนอย               LDH                                <200        200-1,000  >1,000
ประกอบดวย Hodgkin disease, metastatic cancers, primary leiomyosarcomas of the               LDH ratio                          <0.6           >0.6




                                                    AJ
mediastinal vessels, and plasmocytomas.                                                      Protein ratio                      <0.5           >0.5
     Nonmalignant อื่นๆที่เปนสาเหตุของ SVC obstruction ประกอบดวย mediastinal



                                                 IR
                                                                                             Serum pleural-albumin              >1.2           <1.2
fibrosis; vascular diseases เชน aortic aneurysm, vasculitis, and arteriovenous fistulas ;   gradient (serum albumin-



                                IR
infections such as histoplasmosis, tuberculosis, syphilis, and actinomycosis ; benign        pleural albumin)



                               S
mediastinal tumors เชน teratoma, cystic hygroma, thymoma, and dermoid cyst ; cardiac        25.) ตอบ 3. Organophosphate
causes, เชน pericarditis and atrial myxoma; and thrombosis related to the presence of            Organophosphate กระตุน Parasym นะ อาการกคือ SLUDGE 3B
central vein catheters.                                                                           S Sweating + Salivation
24.) ตอบ 4. ใหยาเดิม + ใส ICD                                                                   L Lacrimation
     ขอนี้คิดวาเปนaspiration pneumonia เชื้อที่พบนาจะเปนพวก anaerobe จาก GI tract            U Urination
การใหยา co-amoxyclav ก็สามารถคลุมเชื้อได แตควรใสICDเพื่อระบาย pleural effusion                D Diarrhea
ดวยเพราะเปน complicated pleural effusion (เปน indicationใหใสICD)                             G GI ปวดทอง ( จาก motility เยอะมั้ง)
E Emesis                                                                               31.) ตอบ 2. มีโอกาสเกิด embolic stroke มากกวาคนทั่วไป 15-20%
     B Bradycardia                                                                                Diactolic rumbling murmur นึกถึง MS ซึ่งมักเกิดจาก Rheumatic heart disease
     B Bronchorhea ( secretion เยอะ)                                                        หลังจาก Rheumatic fever / MS ทําใหมีความเสี่ยงตอ embolic stroke มากขึ้น/ ขอ3ผิด
     B Bronchospasm                                                                         เพราะ MS ทําใหเกิด pulmonary hypertension ไดจริง แตเนื่องจากลิ้นหัวใจตีบ ไมใชรั่ว



                                                                                                                                             J
     + miosis + Nicotinic effect เชน M.fasiculation Resp M paralysis +                     32.) ตอบ 3. Bacterial endocarditis



                                                                                                                                          RA
CNS effect เชน seizure Coma CNS depression                                                       ขอนี้ผูปวยมาดวย ไข + HF จึงสงสัย bacterial endocarditis โดยวินิจฉัยตาม Duke




                                                                                                                      RI
26.) ตอบ 1. Botulinum toxin                                                                 criteria (ลองไปหาดูเองนะ) ซึ่งในรายนี้ มี valvular regurgitation, conjuctival hemorrhage,




                                                                                                                   SI
     กตอบขอนี้แหละมีอยูขอเดียว อาการเปนไดตั้งแต หายไดเอง จนถึง เสียชีวิต อาการคือ   fever, Osler’s node(ตุมดําแดงกดเจ็บที่ปลายนิ้ว) ครบ 1 major + 3 minor criteria
symmetrical descending paralysis ,abd cramp ,N/V ,diarrhea ,Resp failure การรักษาคือ              atrial myxoma อาจจะไมมีอาการเลย หรือ มาดวยอาการจาก mass effect หรือ จากการ




                                                                                   AJ
admit observe cli , +-gastric lavage                                                        ที่ tumor ไป involve muscle ปกติ ทําใหเกิด heart failure




                                                                                IR
27.) คือ ขอนี้ มันเปนลักษณะ typical ของ Arsenic poisoning เรยแหละ แลว antidote กคือ           Acute rheumatoid carditis (= acute rheumatic fever) มีอาการไดดังตอไปนี้ carditis,
Dimercaprol (BAL in Oil) หรือ Succimer (DMSA) หรือ Dimerval (DMPS) ไมเห็นมีตํา             polyarthritis, chorea, erythema marginatum, and subcutaneous nodules เปนตน



                                                                              IR
ตอบเรยอะ งง                                                                                      Rupture chordae tendinae ก็จะมีอาการของ mitral valve regurgitation, acute left-



                                                                            S
29.) ตอบ 1. Acute thyroiditis                                                               sided CHF, pulmonary edema, tachypnoea and coughing เปนตน




                                                  AJ
     ผูปวยรายนี้เขาไดกับ Thyrotoxicosis (ภาวะที่รางกายมี thyroid h. มากกวาปกติ) มี    33.) ตอบ 2. Epinephrine
อาการเหนื่อยงาย, ใจสั่น, sweating, tremor, diarrhea, กินเยอะแตน้ําหนักลด; PE : fine             Pulseless Arrest Algorithm : ผูปวยไมมี pulse แตมี electrical activity จึงจัดอยูใน



                                               IR
tremor (hand), lid lag & lid retraction, onycholysis                                        PEA ขั้นตอไปจึง ให epinephrine 1 mg. IV/IO repeat every 3-5 min



                                IR
     พิจารณาลักษณะของกอน เปนได 3 แบบ คือ 1. Diffuse goiter ; 2. Solitary nodule ; 3.     35.) ตอบ 3. Eisenmenger Syndrome



                               S
Multinodular goiter รายนี้เขาไดกับ diffused goiter โดย                                          ซึ่งเปนภาวะที่พบไดในผูปวยที่มี large congenital cardiac หรือ surgically created
     1. Grave’s dis. : thyrotoxicosis, exopthalmos, pretibial myxedema, คลําได thrill      extracardiac left-to-right shunts เปนผลใหเกิดincreased pulmonary blood flowตามมา.
     2. Thyroiditis : แบงเปน acute, subacute, chronic (depend on duration and course of   ตรสจรางกายสามารถพลลักษณะตางๆไดดังนี้ Central cyanosis (differential cyanosis in
disease) คิดวานาจะตอบ acute thyroiditis เพราะมี pain, neck swelling, fever (สวน          the case of a PDA) , Clubbing , Jugular venous pulse (wave may be A-wave dominant,
suppurative thyroiditis นาจะ Unilateral neck pain และจะบวมแดงกดเจ็บบริเวณนั้นดวย)         and, in the presence of a significant tricuspid regurgitation, the V wave may be
     3. อื่นๆ เชน thyrotoxicosis factitious                                                prominent; central venous pressure may be elevated) , Precordial palpation reveals a
     กรณีที่ประวัติแยกไดไมชัดเจนจะสงตรวจ I131 ดูการ uptake เพื่อแยกโรคตอไป              right ventricular heave and, frequently, a palpable S2. , Loud P2 , High-pitched early
diastolic murmur of pulmonic insufficiency , Right-sided fourth heart sound , Pulmonary   38.) ตอบ 5. U/S whole abdomen
ejection click , Single S2                                                                     คิดวาเปน ruptured AAA with decompensated shock Ix ทํา U/S เพื่อ diagnosis and
36.) EKG                                                                                  F/U CT if plan surgery แตถา shock อยูหามทํา ดังนั้น จึงสง U/S whole abdomen
                                                                                          39.) ตอบ 5. NSS



                                                                                                                                           J
                                                                                               ผูปวย shock (tachycardia, BP drop) แบบ hypovolemic (flat neck v.) ตอง resuscitate



                                                                                                                                        RA
Sinus arrest
                                                                                          ดวย isotonic solution คือ NSS




                                                                                                                    RI
                                                                                          40.) ตอบ 4. Stool occult blood (จากชีทอ.กมล recommendation for Thai adults)




                                                                                                                 SI
Sinus bradycardia                                                                              - grade ก.(ควรทํา) แนะนําตรวจ stool occult blood/colonoscopy ทุก 5 ปแนะนําใน
                                                                                          ทุกอายุ ถามีประวัติ family history of colorectal CA, colonic polyp




                                                                                  AJ
                                                                                               - grade ข. (นาทํา) แนะนําตรวจ stool occult blood ทุก 5 ป ที่อายุ> 40 ป จากที่คนนี้




                                                                               IR
                                                                                          ไมมีประวัติครอบครัวจึงนาจะตรวจแค stool occult blood
                                                                                          41.) ตอบ 5. ตรวจ HBs Ag ซ้ําอีก 6 เดือน



                                                                             IR
Complete heart block
                                                                                               ขอนี้มันมีในขอสอบชุดที่ 3 สถาบันจํามา แตไมตรงกันงะ โจทยวา หญิงจะมาสมัคร



                                                                           S
                                                                                          พยาบาล ตรวจเลือดพบ HBsAg + และ HBsAb – (นาจะหมายถึง anti-HBs -ve) สวนชุด




                                                  AJ
                                                                                          ของคณะเราเปน anti-HBc IgM –ve
LBBB                                                                                           การแปลผล คือ HBsAg + แสดงวา มีการติดเชื้อ HBV



                                               IR
                                                                                               anti-HBc IgM –ve แสดงวา ไมใช recent infection



                               IR
                                                                                               anti- HBs -ve แสดงวา ไมมีภูมิ มาตัด choice กัน



                              S
                                                                                               ขอ 1 ให HBV vaccine ผิดแหงๆ เพราะติดเชื้อไปแลว จะมาให vaccine อะไรกัน
Premature atrial beat                                                                          ขอ 2 ให IFN ผิด เพราะจากโจทยบอกวา LFT normal ซึ่งถา ALT ปกติ หรือ
37.) ตอบ 5. C.difficile                                                                   เพิ่มขึ้นนอยกวา 2 เทาของคาปกติ ก็ให follow up ยังไมตองรักษา การ follow up ยังไงก็
     เนื่องจากมีประวัติเขาไดกับ antibiotic associated diarrhea และมีลักษณะเปน mucous   ขึ้นกับวาHBeAg +veหรือ–ve [จะให IFN เมื่อ HBV DNA>20000 หรือ ALT > 2X ULN]
diarrhea                                                                                       ขอ 3 และ 4 ผิดคูกัน เพราะการตรวจ AFP ,U/S liver เปนการตรวจ HCC
                                                                                          surveillance ใน high risk group ซึ่งไดแก ชาย >40 ป , หญิง >50 ป, มี advanced fibrosis/
                                                                                          cirrhosis, มี family Hx of HCC ซึ่งจะตรวจทุก 6 เดือน
จึงตอบ ขอ 5 ตรวจ HBs Ag ซ้ําอีก 6 เดือน ถายังมี HBsAg +ve อยู แสดงวาเปน               Lubricant eg. Mineral oil หลอลื่น แตใขอเสีย คือ ลดการดูดซึมวิตามินที่ละลายใน
HBV carrier เพราะวาสวนใหญรางกายจะสามารถกําจัดเชื้อไปไดเอง                            ไขมัน เพราะมันจะไหลออกมากับอุจจาระเลย หามใชรวมกับ surfactant laxatives
42.) ตอบ 4. delirium tremens                                                                    Note stimulants, lubricant มีการใชเพื่อลดนน. ถือเปน laxatives abuse และมีผล
     เนื่องจากผูปวยมีประวัติดื่มเหลามานาน และหลังเขารักษาในรพ. แพทยไดNPOทันที       รบกวน normal bowel habit



                                                                                                                                             J
จึงเกิดอาการอาละวาดดึงสายน้ําเกลือ                                                              Surfactant laxatives eg. castor oil ทําให stool นุมออกงายขึ้น castor oil ใชเตรียม



                                                                                                                                          RA
     อาการDT ไดแก confusion, diarrhea, disorientation and agitation, perceptual         bowel เพราะออกฤทธิ์เร็ว (1-3 hr) แตมีขอเสียคือ electrolyte imbalance ได




                                                                                                                     RI
disturbance(visual), autonomic hyperactivity (high pulse, blood pressure, and rate of           Saline and Osmotics laxatives eg. MOM ; milk of Magnesia, Unison suppository




                                                                                                                  SI
breathing)                                                                                (Na) ออกฤทธิ์เร็วใน 3 hr ทําให osmolarity in colon สูงขึ้นจึงดึงน้ําออกมาคลุกเคลากับ
     สาเหตุ: ผูปวยมักมีประวัติดื่มalcoholมานาน และมีการหยุดดื่มทันที ทําใหเกิดอาการ    อุจจาระ ซึ่งมันก็จะดึงน้ําออกมาพรอมกับ electrolyte ดวย ดังนั้นจึงมีขอเสีย คือ electrolyte




                                                                                 AJ
withdrawal Delirium tremens มักเกิด3-7 วัน หลังการดื่มครั้งสุดทาย หากไมไดรับการ        imbalance และอาจมีการดูดซึม Mg เขากระแสเลือดได เกิด SE ของ Mg, ไมควรใชใน




                                                                              IR
รักษาอาจทําใหเสียชีวิตได(mortality rate35%) สําหรับผูปวยที่มีความเสี่ยงสูงไดแก      ผูปวยโรคไต
extreme fever, fluid and electrolyte imbalance, or intercurrent illness such as occult    44.) ตอบ 1. ENT exam



                                                                            IR
trauma, pneumonia, hepatitis, pancreatitis,alcoholic ketoacidosis,Wernicke-korsakoff            neck mass ที่ นานกวา 2 wk ควร refer เพื่อตรวจ ENT exam ทุกราย



                                                                          S
syndrome                                                                                  45.) ตอบ 2. Plasmodium vivax




                                                 AJ
     การรักษา : ใหยาในกลุมbenzodiazepines                                                                   P. falciparum        P. vivax            P. malariae         P. ovale
43.) ตอบ 1. bulk forming agent                                                            Form ที่พบได - Ring form                พบไดทุกระยะ พบไดทุกระยะ พบไดทุกระยะ



                                              IR
     ยาระบาย (laxatives) มีดังนี้                                                         ใน peripheral (Trophozoites)



                               IR
     Fiber supplements or bulk laxatives eg.Metamucil เปน laxatives ที่ปลอดภัยที่สุด     blood smear - Gametocyte



                              S
กลไกคือ fiber จะดูดน้ําทําใหอุจาระขยายขนาดใหญขึ้นไปกระตุน reflex ใน colon ดังนั้น      ลักษณะของ - normal                       - ขนาดใหญ สี - ขนาดปกติ                - ขนาดปกติ
จึงตองกินน้ําตามเยอะๆ ถากินน้ําไมพอก็จะอุดตันลําไสได ขอเสียคือใชเวลาหลายวันกวา    RBC                 size&shape           จางลง               หรือเล็กกวา        หรือใหญ สีจาง
จะไดผล และพวก fiber จะลดการดูดซึมของยาและวิตามิน                                                             - Maurer’s dots - Schuffner’s ปกติ                           ลง
     Stimulants eg. Bisacodil (Dulcolax), Senokot กระตุน n.plexus ในลําไสทําให colon                                            dots                - ziemann’s         - tear drop RBC
m. หดตัว(กระตุน reflex โดยตรง) แตถาใชนานๆ n.plexus จะถูกทําลายทําใหตอมาทองผูก                                                                   dots                - Schuffner’s
ไปเลย จึงไมควรสั่งใหคนไขที่ทองผูกเปนประจําใช                                                                                                                         dots
                                                                                          จํานวน              20 (16-32)           16 (12-24)          8 (6-12)            8 (6-12)
merozoites                                                                                46.) ตอบ 4. artisunate + mefloquine
ใน RBC                                                                                    47.) ตอบ 1. Acyclovir
รูปรางของ       Crescent form       Spherical        Spherical         Spherical shape             ขอนี้หลัก ๆ คือใหดู CSF profile ซึ่งพบวา glucose และ protein ไมตางจากคา
Gametocyte                           shape            shape                               ปกติมากนัก และ wbc ก็มี Lymphocytosis ถึง 95% (มันไมมีคา wbc มาให แตคิดวาคงไม



                                                                                                                                          J
                                                      (มี growing                         สูงมาก) G/S ก็ไมพบ จึง R/O bacteria ออกไปไดเลย สวน fungus , TB มี lymphocytosis



                                                                                                                                       RA
                                                      trophozoites                        ได แตควรมีการเปลี่ยนแปลงของระดับ glucose และ protein ที่ชัดเจนกวานี้ จึงคิดถึงนอย




                                                                                                                   RI
                                                      เปน band                           (Indian ink ก็ไมเจอ crypto , choice ก็ไมมี TB) นาจะเปน viral มากที่สุดนะจะ




                                                                                                                SI
                                                      form)                               48.) ตอบ ไมมีรูปอะ เลยไมรูจะเฉลยอะไรแตจะสรุปใหแลวกันนะ
Complication cerebral malaria                                                                 ลักษณะ           P.falciparum          P.vivax         P.malariae          P.ovale




                                                                                 AJ
Treatment                                                                                 infected rbc      ปกติ               ใหญ ติดสีจาง ปกติหรือเล็ก         ใหญ,รี.




                                                                              IR
1. P. falciparum                                                                                                                                                  fimbriation
     - with complication :



                                                                            IR
                                                                                          ระยะของเชื้อที่ มักพบเฉพาะ           พบทุกระยะ        พบทุกระยะ         พบทุกระยะ
          - Quinine dihydrochloride 20 mg/kg IV drip in 4 hr (loading dose) then          ตรวจพบ          ring form


                                                                          S
     10mg/kg IV drip in 2 hr q 8 hr for 7 days                                            Ring form       1/6-1/5 of rbc       1/3 of rbc       1/3 of rbc        1/3 of rbc



                                                    AJ
           - Artesunate 2.4 mg/kg IV then 1.2 mg/kg q 12 hr จนครบ 600 mg                                  วงบางมาก             วงบาง            วงหนา             วงคอนขางหนา




                                                 IR
     - no complication :
                                                                                                          - 2 chromatins
          - Quinine sulfate 600 mg oral q 8 hr + Tetracycline 250 mg oral q 6 hr for 7



                               IR
                                                                                                          - multiple
     days (ในเด็กให quinine sulfate 10 mg/kg oral q 8 hr ใน 4 วันแรก then 15-20 mg/kg
                                                                                                          infection


                              S
     q 8 hr for 4 days)
                                                                                                          - marginal
          - Artesunate 100 mg oral then 50 mg oral q 12 hr จนครบ 600 mg then
                                                                                                          form
     Mefloquine 750 mg oral then 500 mg oral หลัง dose แรก 6 hr
                                                                                          Growing         เหมือน ring          คลายอะมีบา      Band form    Cytoplasm เปน
2. P. vivax & P. ovale
                                                                                          trophozoite     form                 Pseudopod        compact form วงหนา,
     - Chloroquine + Primaquine oral for 14 days (หากเปน G-6-PD จะลด dose
                                                                                                                               มาก                           pseudopod
primaquine)
                                                                                                                               Vacuole ชัด                   นอย. Vacuole
3. P. malariae - Chloroquine 600 mg (10 mg/kg ในเด็ก) oral
ไมชัด             สาเหตุ แต the most common cause of erythema nodosum คือ Bacterial infections
Schizont          8-32(24)            12-24(16)         6-12(8),          4-12(8)            โดยเฉพาะ Streptococcal infections พบทั้งในเด็กและผูใหญ ในผุใหญที่พบบอยอีกอันคือ
                                                        rosette                              sarcoidosis เพราะThe most common cutaneous manifestation of sarcoidosis is erythema
Gametocyte          Crescent            กลม             กลม                กลม               nodosumนั่นเอง นอกจากนี้ infectionsในผูใหญ(โดยเฉพาะใน developing countries อยาง



                                                                                                                                             J
                                                                                             ประเทศเรา) ตองคิดถึง Tuberculosisดวย(จริงๆยังมีอีกหลายเชื้อเลย สามารถคนดูใน



                                                                                                                                          RA
49.) ตอบ 1. branching septate hyphae
     Tinea cruris (ขาหนีบ) การติดเชื้อราในกลุม dermatophytes หรือ กลาก ลักษณะ               หนังสือ หรือ emedicineได)




                                                                                                                      RI
lesion เปน erythematous active border with central clearing, scaly (ขุยๆ) pruritic (คันๆ)        ดังนั้น investigationในรายนี้ คือการหาการติดเชื้อนั่นเอง ซึ่งอาจทําไดหลายอยาง เชน




                                                                                                                   SI
Ix คือ KOH พบ branching septate hyphae with arthrospore Rx คือ clotrimazole ทา bid           throat culture,stool examination, blood cultures ตามเชื้อที่สงสัย ซึ่งในผูใหญเราสงสัย
2-3 wk (Tinea ใชยาทาก็พอ ยกเวน Tinea capitis (หนังหัว) และ Tinea unguim (เล็บ) ตอง        Tuberculosis กับ sarcoidosis ดังนั้นตองสง film chest x-ray ดวย จึงตอบขอ 1




                                                                                    AJ
ใหยากินดวย เชน griseofulvin, itraconazole)                                                51.) ตอบ 4. Praziquantel (จากภาพเปนไขของ Opisthorchis viverrini)




                                                                                 IR
     Pseudohyphae with budding yeast = Candida                                               52.) ตอบ 2. Euglobulin test
                                                                                                  Bleeding time I/C : ผูปวยที่มีเลือดออกผิดปกติ ที่นาจะเกิดจาก primary



                                                                               IR
     Short septate hyphae with oval yeast (spaghetti and meatball) = Malassezia furfur
(โรคเกลื้อน Pityriasis versicolor)                                                           hemostatic defect แตมีจํานวนเกร็ดเลือดปกติ



                                                                             S
50.) ตอบ 1. CXR                                                                                   patient preparation :




                                                     AJ
     จากโจทย นึกถึง Erythema nodosum (EN) มากที่สุดเนื่องจากอาการแสดงของผูปวย                  - แขนของคนไขขางที่จะตรวจ ตองไมไดอยูระหวางการได IV fluid ใดๆ ทั้งสิ้น




                                                  IR
เขาไดdefinitionของโรคนี้เลย คือ                                                                 - คนไขที่รับประทานยา Aspirin ตองหยุดยา 7 วันขึ้นไป กอนรับการตรวจ
     Erythema nodosum (EN) is an acute, nodular, erythematous eruption that usually               - ไมตรวจในผูปวยที่มี Hct ต่ํากวา 30 %



                                IR
is limited to the extensor aspects of the lower legs.                                             วิธีการวิเคราะห (methodology) : Modified Ivy method โดยควบคุมความดันเลือด



                               S
     โดยมีclinical presentation คือ The eruptive phase of erythema nodosum begins            ของแขน ผูปวยดวย เครื่องวัดความดันโลหิต ที่ 40 มิลลิเมตรปรอท และใชมีดปลายแหลม
with flulike symptoms of fever and generalized aching ก็คือมี Febrile illness with           เจาะผิวหนังใหเปนแผลลึก 3 มิลลิเมตร กวาง 2 มิลลิเมตร จับเวลาตั้งแตเลือดเริ่มไหลจน
dermatologic findings includes abrupt onset of illness with initial fever, followed by       เลือดหยุดไหล โดยใช กระดาษกรองซับเลือด
a painful rash within 1-2 days. ก็เหมือนในโจทยนั่นเอง                                       ----> ผูปวย Dengue มี thrombocytopenia อยูแลว
     โดยโรคนี้เปน delayed hypersensitivity reaction and may occur in association with            Euglobulin lysis time I/C : ผูปวยที่สงสัยภาวะ increased fibrinolysis
several systemic diseases or drug therapies, or it may be idiopathic. โดยมีสาเหตุไดหลาย     ----> Hemostasis depends on the balance between coagulation and fibrinolysis
                                                                                                  Laboratory tests for assessment of the coagulation system
- Coagulation Screening tests , Activated partial thromboplastin Time ,                             เชื้อ Naegleria fowleri เขาสูรายกายทางโพรงจมูก มักจะเกิดเมื่อมีอากรสําลักน้ํา และ
Prothrombin time , Thrombin time , Fibrinogen , Fibrinolysis , Screening tests ,                    เชื้อจะขึ้นสมองโดยผานทางเสนประสาทรับกลิ่น (olfactory nerve) ทําใหเกิดเยื้อหุมสมอง
Euglobulin clot-lysis time , Dilute-whole-blood clot-lysis time                                     อักเสบ (Primary amoebic meningo-encephalitis หรือ PAM) N. fowleri มีรูปรางลักษณะ 2
      ผลแทรกซอนของโรค Dengue                                                                       ระยะ ไดแกระยะ trophozoite ซึ่งอาจมีรูปรางแบบอะมีบา (ameboid form) กับแบบมี



                                                                                                                                                    J
            1. Prolonged shock                                                                      หนวดสําหรับวายน้ํา (flagellate form) และระยะ cyst เชื้อเจริญเติบโตไดดีที่อุณหภูมิสูง



                                                                                                                                                 RA
            2. Fluid overload                                                                       ระยะฟกตัวของโรค 3-7 วัน ผูปวยจะมีอาการปวดศีรษะมาก เจ็บคอ คลื่นไส อาเจียน




                                                                                                                              RI
            3. Massive bleeding                                                                     เพอ ไขสูง คอแข็ง งวงซึม อาการจะทรุดอยางรวดเร็ว และมักจะเสียชีวิตภายใน 10 วัน




                                                                                                                           SI
            4. Encephalopathy จาก hepatic encephalopathy                                            โรคนี้สวนใหญพบในคนวัยหนุมสาวทีมีสุขภาพ
53.) ตอบ 1. Melioidosis                                                                             56.) ตอบ 2. Chikungunya




                                                                                          AJ
      เนื่องจาก fever 2wk, FBS 260 (นาจะมี U/D เปน DM), BUN/Cr สูง (อาจมี renal                         ติดตอโดยยุงลาย Aedes aegypti อาการไดแก ไขสูงเฉียบพลัน ผื่นแดงตามรางกาย อาจ




                                                                                       IR
impairment อยูเดิม), ตับโต มามโต ; u/s พบ hypoechoic ก็นาจะบงถึง abscess ซึ่งมี                 คันรวมดวย conjunctival injection,migratory polyarthritis โดยมักเปนที่ขอเล็กๆ เชน
multiple ที่มาม ; จากประวัติทั้งหมดนี้เปน typical natural Hx of “ Melioidosis ”                   ขอมือ ขอเทา อาการหายภายใน 1-2 wk ไมรุนแรงถึงshock อาจพบpetechiae หรือ



                                                                                     IR
      Hx : ผูปวยมี U/D เชน DM, renal fail, thalassemia อาชีพที่สัมผัสดิน เชนทํานา ตรวจ          tourniquet +ได ขอแตกตางจาก Dengue คือ ไขสูงเฉียบพลันกวา และระยะไขเพียง 2 วัน,



                                                                                   S
พบบาดแผลที่ขา/เทา มาพบแพทยโดยไขสูง                                                               จํานวนpetechiaeใน tourniquet testนอยกวา,ไมพบconvalescent rash,พบMP




                                                       AJ
      Ix : พบ abscess ตาม organ ตางๆ ที่พบบอย : liver, spleen, skin                               rash,conjunctival infection,myalgia,arthalgiaไดบอยกวา,ชักจากไข15%ซึ่งมากกวา
      Rx :1. Acute : Ceftazidime iv or Imipenem or Meropenem นาน 2 wks                              dengue3เทา ระบาดชวงฤดูฝน เปนไดทุกกลุมอายุ รักษาโดย supportive treatment



                                                    IR
         2. ตอมา : Co-trimoxazole + Doxycline (ในเด็กให Augmentin แทน) นานรวม                           Rickettsial infection แบงเปน3กลุมไดแก spotted fever,typhus,scrub typhus อาการ



                                   IR
ระยะ acute = 20 wks                                                                                 เดนคือไขสูงหลายวัน อาการอื่นๆ เชน ปวดหัว ปวดกลามเนื้อ ออกผื่น คลื่นไส อาจมี



                                  S
54.) ตอบ 3. Strongyloides stercolaris                                                               eschar diag โดย serology รักษาโดย doxycycline
      ผูปวยที่มีการติดเชื้อ stongyloid อยูกอน เมื่อไดยา steroids ซึ่งกดภูมิคุมกันทําใหเกิด         Rheumatic heart diseaseลิ้นหัวใจถูกทําลายจากกระบวนการเกิดโรคซึ่งเริ่มตนจาก
disseminated strongyloidiasis คือแพรกระจายไปหลาย system ผูปวยอาจมาดวย                           การเจ็บคอจากการติดเชื้อ streptococcus เปนการดําเนินโรคหนึ่งของ RF acuteมักมาดวย
abdominal pain, distension, shock, pulmonary and neurologic complications, septicemia               carditis(ไมคอยมีผลตอ hemodynamic) chronicมักมาดวยmitral stenosis
ดังนั้นจึงควรตรวจอุจจาระหา larva ของมันกอนเริ่มยาเสมอ
55.) ตอบ 1. N. fowleri
57.) ตอบ 2. xertic eczema                                                                                ‐ ถาไมเสี่ยง ให NG หรือ PEG โดยถาตองใหนานเกิน 1 เดือน พิจารณาใหทาง
      อาจเรียก asteatotic eczema หรือคนทั่วไปเรียก winter itch พบมากในผูสูงอายุ                              PEG
โดยเฉพาะชวงอากาศหนาวหรือแหงกวาปกติ สามารถพบบริเวณหนาแขงไดบอย                                เนื่องจากรายนี้ ทางเดินอาหารใชได แตกินอาหารเองไดนอย มีความเสี่ยงต่ําในการ
เนื่องจากเปนบริเวณที่มี subcutaneous tissue นอยกวาสวนอื่นๆ ของรางกาย                     aspirate ดังนั้นสามารถใหผาน NG / PEGได แตเนื่องจากรายนี้นาจะตองใหอาหารทางนี้



                                                                                                                                               J
58.) ตอบ ?                                                                                    เกิน 1 เดือนดังนั้น นาจะใหทาง PEG



                                                                                                                                            RA
      Zinc deficiency: Abnormal Growth, Hypogonadism, Skin Problems, Weight Loss,             61.) ตอบ 5. ควบคุมอาหาร + ลดน้ําหนัก




                                                                                                                        RI
Diarrhea, Emotional Instability, Hair Loss                                                         เนื่องจาก primary hypertension ควรใหการรักษาดวย life-style modifications (weight




                                                                                                                     SI
      Retinol deficiency: Bitot spots, nyctalopia, Dry skin, Dry hair, Pruritus, Broken       loss, smoking cessation, salt reduction) กอนหากยังไมสามารถควบคุมระดับความดัน
fingernails, Keratomalacia, Xerophthalmia, Follicular hyperkeratosis                          โลหิตไดตามเปาหมายจึงจะพิจารณาใหยา anti-hypertensive drugs ตอไป




                                                                                      AJ
      Selenium deficiency:keshan disease, Kashin-Beck disease,(myocardial necrosis,                                                 Pediatrics



                                                                                   IR
atrophy, degeneration and necrosis of cartilage tissue), hypothyroidism                       1.) ตอบ 2. พาไปพบจิตแพทย
59.) ตอบ 1. thiamine



                                                                                 IR
                                                                                                   คิดถึง ADHD (Attention deficit hyperactive disorder) มี 3 กลุมอาการหลัก กอน
      Wet beriberi (cardiac beriberi) โดยพบในผูที่ไดรับวิตามิน บี1 นอยกวา 0.2             อายุ 7 ป ( แตเด็กคนนี้ 8 ป อานะ) คือ 1. ซน (Hyperactive) , 2. ขาดสมาธิ (Inattentivit) ,


                                                                               S
มก./1,000 กิโลแคลอรี่ มีอาการสําคัญคือบวม ระยะแรกจะรูสึกขาหนัก เดินลําบาก บวม                3. หุนหันพลันแลน (Impulsivity)



                                                    AJ
เล็กนอยที่ขาและหนา เนื่องจากมี lactic acid และ pyruvic acid คั่ง เพราะ oxidative                 Rx. Medication (stimulant;Methylphenidate) + Behavior modification




                                                 IR
decarboxylation นอยลง ทําใหเซลลอักเสบและมีสารน้ํารั่วออกมา และหัวใจขาดพลังงาน                   Impact . การเรียน สังคม การปรับตัว self esteem
เพราะใชสารทั้งสองไดลดลง



                                 IR
                                                                                              2.) ตอบ 3. stranger anxiety
60.) ตอบ 4. PEG (Percutaneous endoscopic gastrostomy)                                                    เพราะเด็กอายุ 9 mo นาจะมี stranger anxiety เพราะฉะนั้นมักจะกลัวคนแปลก


                                S
      เรื่อง nutrition ตองประเมินกอนวา GI function ไดหรือไม ถาได ก็ดูวากินเองไดมาก   หนา ไมไดยอมใหทุกคนอุม จึงคิดวาขอนี้เปนพัฒนาการที่ผิดปกติสําหรับเด็ก 9 mo
นอยแคไหน                                                                                               เด็กอายุ 9 mo : Gross&fine motor -> pull to stand,peek- a- boo (จะเอ), mature
      - กินไดเองมากกวา 75% ใหกินเอง                                                        pincer grasp, นั่งไดมั่นคง, คลาน / Language-> immature jagoning / Social-> จะเอ,
      - กินได 50-75% ใหกินเองและ oral supplement                                            object permanent / Stranger anxiety เริ่มตอนอายุ 6-9 mo ถึง 2 yr เดนตอนอายุ 9 mo
      - กินไดเองนอยกวา 50% ให enteral feeding                                             3.) ตอบ 4. ลดนมเหลือวันละ 2 มื้อ
      ซึ่ง enteral feeding ตองพิจารณาวามีความเสี่ยงตอ aspiration ไหม                            เด็ก 2-6 ขวบ คิดน้ําหนักโดย 2x+8 คนนี้ 2ขวบ ได 12 Kg น้ําหนักเกินนะ เพราะหนัก
             ‐ ถาเสี่ยง ให ND/ NJ tube
16 Kg สวนสูง ใช 6x+77 กอเกือบพอดีเลย -> คิดวาขอนี้นาจะตอบไปในทางลด จะลดมือ             ที่ไมตองใชทักษะฝมือ หรือกึ่งใชฝมือ แตอาจตองการคําแนะนํา และการชวยเหลือบางเมื่อ
อาหารก็ไมได เพราะเด็กจะไดรับสารอาหารไมครบ แตถาลด นมก็พอได                             ประสบความเครียด
4.) ตอบ 3. รับ OPV, DTP                                                                            IQ 35-49 moderate MR สามรถเรียนจบ ป.1-ป.2 สามารถฝกอบรมได (trainable) ใน
                                                                                             ทักษะการชวยเหลือ ดูแลตนเอง เรียนรูที่จะเดินทางไดดวยตนเองในสถานที่ที่คุนเคย และ



                                                                                                                                              J
Birth 1mo         2mo        4mo        6mo      9mo 12mo         18mo       4-6yr     >4-   ฝกอาชีพไดบาง สามารถทํางานที่ไมตองใชทักษะฝมือ แตควรอยูภายใตการกํากับดูแล



                                                                                                                                           RA
                                                                                       6yr   อยางใกลชิด




                                                                                                                       RI
BCG                                                                                                IQ20-34พอจะฝกฝนทักษะการดูแล ตนเองเบื้องตนไดบางแตนอย ดํารงชีวิตอยูใน




                                                                                                                    SI
HBV1         HBV2                      HBV3                                                  สังคมภายใตการควบคุมดูแลอยางเต็มที่ การทํางานตองการโปรแกรมในชุมชน หรือการ
                                                                                             ใหความชวยเหลือที่พิเศษเปนการเฉพาะ
                DTP1& DTP2& DTP3&                                DTP4& DTP5&



                                                                                   AJ
                                                                                                  IQ <20 พัฒนาการลาชาอยางชัดเจนในทุกๆดาน มักมีพัฒนาการดานการเคลื่อนไหว
                OPV1 OPV2 OPV3                                   OPV4 OPV5




                                                                                IR
                                                                                             และฝกการชวยเหลือตนเองไดบาง มีขีดจํากัดในการเขาใจและการใชภาษาอยางมาก
                                                    MMR1                    MMR2             ตองการความชวยเหลือ ดูแลอยางใกลชิดตลอดเวลา



                                                                              IR
                                                                 JE1&2                       7.) ตอบ 3. Amblyopia



                                                                            S
                                                                 หางกัน                           จากโจทยผูปวยมีภาวะ misalignment ของ rt eye (internal strabismus) complication ที่




                                                   AJ
                                                                   1-4                       อาจเกิดขึ้นไดคือ เนื่องจากวาเด็กนั้นนารัก มีการปรับตัวอยางดีเยี่ยม พอตา 2 ขางมองได
                                                                                             ไมเหมือนกัน จากอะไรก็ตาม... สมองก็เลยมีการปรับใหตาที่มีปญหานั้นลดการมองเห็นลง



                                                IR
                                                                 สัปดาห
                                                                                       DT    ไป เลยกลายเปนตาขี้เกียจ amblyopia นั่นเอง



                                IR
คําอธิบาย Expanded program of immunization (EPI)                                                   Key อยูที่วา เราตอง detect กอนอายุ 7-8 ป แลวรีบรักษาซะ เพราะถาเปนแรวจะ รักษา



                               S
5.) ตอบ 1. TORCH titer                                                                       ไมได
     เพราะ เด็กมี increase muscle tone , hyperreflexia , delay development (6เดือน ยังชัน           Amblyopia หมายถึงภาวะที่ตามี visual acuity ลดลงโดยไมมี organic lesion
คอไมได) head circumference มากผิดปกติ ทําใหนึกถึง involve CNS จึงคิดถึง TORCH             ถาตาขางหนึ่งมี VA ที่แกไขแลว (ดวย pinhole หรือแวน) นอยกวาตาอีกขางหนึ่ง เทากับ
infection                                                                                    2 แถวของ Snellen Chart ขึ้นไป
6.) ตอบ 2.ออกใบรับรองเขาเรียนไดโดยมีอาจารยพิเศษ                                           8.) ตอบ 1. Viral croup
     IQ 50-69 (60) เปน mild MR สามารถเรียนรูได (educable) สามารถเรียนจนจบชั้น                   Viral croup (laryngotracheobronchitis) common ในเด็กอายุ 6 เดือน – 4 ป
ประถมปลายได สามารถฝกทักษะดานสังคมและอาชีพ พอที่จะเลี้ยงตัวเองได เปนแรงงาน
เปน Inflammation and edema of the subglottic area causing airway obstruction in the            11.) ตอบ 3. CXR PA, lateral
larynx, trachea, or bronchi. ตรวจรางกายมี inspiratory stridor มี sign ของ upper airway               เด็กมี inspiratory stridor ซึ่งเปนลักษณะของ upper airway obstruction และมี rhonchi
obstruction ; สาเหตุสวนใหญเกิดจากเชื้อไวรัส(Parainfluenza virus) แตก็อาจเกิดจาก              ซึ่งเปนลักษณะของ lower respiratory tract obstruction โรคที่ทําใหเกิดลักษณะเชนนิ้ คือ
bacterial infection ได เชน S. aureus, Mycoplasma pneumoniae, S. pneumoniae, S.                Croup = Viral laryngotracheobronchitis (larynx = upper, bronchus = lower) การตรวจ X-



                                                                                                                                                J
pyogenes, Hib (bacterial croup จะ present ดวย high fever & more severe resp. distress)         ray ที่ neck AP พบ pencil sign, steeple sign Lateral พบ ballooning of hypopharynx จึง



                                                                                                                                             RA
     acute epiglottitis มักเกิดในเด็กอายุ 2-7 ป ทําใหเกิด supraglottic stenosis มี sign ของ   ตอบ CXR PA,lateral เพราะสามารถดูสวนคอไดดวย




                                                                                                                         RI
upper airway obstruction ไดเชนกัน และลักษณะ stridor เปน inspiratory stridor ผูปวยจะ        12.) ตอบ 3. SVC obstruction




                                                                                                                      SI
มีไขสูง, muffle voice + 4Ds symptoms (drooling, dyspnea, dysphagia, dysphonia)                       classic symptoms and signs become more obvious. Dyspnea is the most common
     Whooping cough (pertussis) เปนการอักเสบของทางเดินหายใจสวน bronchus มัก                   symptom , Other symptoms include facial swelling, head fullness, cough, arm swelling,




                                                                                       AJ
เปน expiratory stridor มีอาการไอมาก อาจตรวจพบ subconjuctival hemorrhage, rectal                chest pain, dysphagia, orthopnea, distorted vision, hoarseness, stridor, headache, nasal




                                                                                    IR
prolapsed ได lung มี croase crepitation, ไมคอยตรวจพบวามี tachypnea การตรวจ CXR              stuffiness, nausea, pleural effusions, and light-headedness.
อาจพบ peribronchial thickening                                                                        findings : venous distension of the neck and chest wall, facial edema, upper



                                                                                  IR
 9.) ตอบ 3. Retropharyngeal abscess                                                             extremity edema, mental changes, plethora, cyanosis, papilledema, stupor, and even



                                                                                S
     ตรวจรางกายพบ Bulging posterior pharynx                                                    coma.




                                                     AJ
10.) ตอบ 2. ICD                                                                                 13.) ตอบ 2. Acute Renal Failure
     ผูปวยนาจะมี Pneumonia with parapneumonic effusion เยอะพอสมควร trachea ถึง                     จากโจทย นาจะเปน MAHA ที่พบบอยในเด็กคือ HUS โดยจะมีอาการนําคือ มีไข



                                                  IR
จะshift ไมแนใจนาจะ 2. ICD นะ เพราะถา 1. mycoplasma titer จาก mycoplasma                     ถายปนมูกเลือด โรคนี้เปน vascular-occlusive disorder จึงมีการแตกของเม็ดเลือดแดง



                                 IR
pneumomiae จะเปนแบบ walking pneumonia คืออาการไมคอยมาก แต CXR ดู severe อะ                 เกล็ดเลือดต่ําได & จาก U/A แสดงวานาจะมี intravascular hemolysis ก็ควรระวัง Acute



                                S
ก็พบวาทําใหเกิด pleural effusionได แต rare อะ อาการของ mycoplasma จะ non-specific          Renal Failure
onset - insidious, with fever, malaise, headache, and cough. Cough is a hallmark of M           14.) ตอบ 4. ANA
pneumoniae infection และก็มี extrapulmonary manifestation ได เชน มีผื่น maculo                      เหมือนมี multiorgan involvement ( Hematology, mucosal involvement , kidney
papular, erythema multiforme, bullous myringitis, musculoskeletal เชน ปวดขอ etc. [            involvement) เหลืออีกหนึ่งอยางจะครบ criteria SLE แลว ก็ตรวจANA ไปเลยดิจะได
nasopharyngeal swab ไมรูนะวาขอนี้เคาตั้งใจจะใหเปน Flu เพราะชวงนั้น Fluระบาด]            วินิจฉัย
Nt2009 Complete Ans
Nt2009 Complete Ans
Nt2009 Complete Ans
Nt2009 Complete Ans
Nt2009 Complete Ans
Nt2009 Complete Ans
Nt2009 Complete Ans
Nt2009 Complete Ans
Nt2009 Complete Ans
Nt2009 Complete Ans
Nt2009 Complete Ans
Nt2009 Complete Ans
Nt2009 Complete Ans
Nt2009 Complete Ans
Nt2009 Complete Ans
Nt2009 Complete Ans
Nt2009 Complete Ans
Nt2009 Complete Ans
Nt2009 Complete Ans
Nt2009 Complete Ans
Nt2009 Complete Ans
Nt2009 Complete Ans
Nt2009 Complete Ans
Nt2009 Complete Ans
Nt2009 Complete Ans
Nt2009 Complete Ans
Nt2009 Complete Ans
Nt2009 Complete Ans
Nt2009 Complete Ans
Nt2009 Complete Ans
Nt2009 Complete Ans
Nt2009 Complete Ans
Nt2009 Complete Ans
Nt2009 Complete Ans
Nt2009 Complete Ans
Nt2009 Complete Ans
Nt2009 Complete Ans
Nt2009 Complete Ans
Nt2009 Complete Ans
Nt2009 Complete Ans
Nt2009 Complete Ans
Nt2009 Complete Ans
Nt2009 Complete Ans
Nt2009 Complete Ans

More Related Content

What's hot

Nt2553step3round1 28NOV2553
Nt2553step3round1 28NOV2553Nt2553step3round1 28NOV2553
Nt2553step3round1 28NOV2553vora kun
 
Osce ศรว ครั้งที่สอง 10jan53
Osce ศรว ครั้งที่สอง 10jan53Osce ศรว ครั้งที่สอง 10jan53
Osce ศรว ครั้งที่สอง 10jan53vora kun
 
NTstep3round2 9_jan2554
NTstep3round2 9_jan2554NTstep3round2 9_jan2554
NTstep3round2 9_jan2554vora kun
 
Step3 Tutorial by SWU book1
Step3 Tutorial by SWU book1Step3 Tutorial by SWU book1
Step3 Tutorial by SWU book1vora kun
 
Compre si 2010 ans
Compre si 2010 ansCompre si 2010 ans
Compre si 2010 ansvora kun
 
คู่มือเวชปฏิบัติหัตถการสำหรับ นศพ
คู่มือเวชปฏิบัติหัตถการสำหรับ นศพคู่มือเวชปฏิบัติหัตถการสำหรับ นศพ
คู่มือเวชปฏิบัติหัตถการสำหรับ นศพFone Rati
 
สอบ-ศรว-มีนาคม-2551
สอบ-ศรว-มีนาคม-2551สอบ-ศรว-มีนาคม-2551
สอบ-ศรว-มีนาคม-2551rookiess
 
หัตถการที่จำเป็นทางสูติ
หัตถการที่จำเป็นทางสูติหัตถการที่จำเป็นทางสูติ
หัตถการที่จำเป็นทางสูติvora kun
 
การซักประวัติการเจ็บป่วย
การซักประวัติการเจ็บป่วยการซักประวัติการเจ็บป่วย
การซักประวัติการเจ็บป่วยOzone Thanasak
 
11แผน
11แผน11แผน
11แผนFmz Npaz
 
Case study surgery
Case study surgeryCase study surgery
Case study surgerysoftmail
 
Osce คณะ si 115
Osce คณะ si 115Osce คณะ si 115
Osce คณะ si 115vora kun
 
แนวทางเวชปฏิบัติในการรักษาผู้ป่วยมาลาเรียในประเทศไทย พ.ศ.2557
แนวทางเวชปฏิบัติในการรักษาผู้ป่วยมาลาเรียในประเทศไทย พ.ศ.2557แนวทางเวชปฏิบัติในการรักษาผู้ป่วยมาลาเรียในประเทศไทย พ.ศ.2557
แนวทางเวชปฏิบัติในการรักษาผู้ป่วยมาลาเรียในประเทศไทย พ.ศ.2557Utai Sukviwatsirikul
 

What's hot (19)

Nt2553step3round1 28NOV2553
Nt2553step3round1 28NOV2553Nt2553step3round1 28NOV2553
Nt2553step3round1 28NOV2553
 
Osce ศรว ครั้งที่สอง 10jan53
Osce ศรว ครั้งที่สอง 10jan53Osce ศรว ครั้งที่สอง 10jan53
Osce ศรว ครั้งที่สอง 10jan53
 
NTstep3round2 9_jan2554
NTstep3round2 9_jan2554NTstep3round2 9_jan2554
NTstep3round2 9_jan2554
 
Step3 Tutorial by SWU book1
Step3 Tutorial by SWU book1Step3 Tutorial by SWU book1
Step3 Tutorial by SWU book1
 
Example osce
Example osceExample osce
Example osce
 
Key word osce
Key word osceKey word osce
Key word osce
 
Nle step 2_2553
Nle step 2_2553Nle step 2_2553
Nle step 2_2553
 
Compre si 2010 ans
Compre si 2010 ansCompre si 2010 ans
Compre si 2010 ans
 
คู่มือเวชปฏิบัติหัตถการสำหรับ นศพ
คู่มือเวชปฏิบัติหัตถการสำหรับ นศพคู่มือเวชปฏิบัติหัตถการสำหรับ นศพ
คู่มือเวชปฏิบัติหัตถการสำหรับ นศพ
 
สอบ-ศรว-มีนาคม-2551
สอบ-ศรว-มีนาคม-2551สอบ-ศรว-มีนาคม-2551
สอบ-ศรว-มีนาคม-2551
 
Case study : dengue fever
Case study : dengue feverCase study : dengue fever
Case study : dengue fever
 
หัตถการที่จำเป็นทางสูติ
หัตถการที่จำเป็นทางสูติหัตถการที่จำเป็นทางสูติ
หัตถการที่จำเป็นทางสูติ
 
การซักประวัติการเจ็บป่วย
การซักประวัติการเจ็บป่วยการซักประวัติการเจ็บป่วย
การซักประวัติการเจ็บป่วย
 
11แผน
11แผน11แผน
11แผน
 
Case study surgery
Case study surgeryCase study surgery
Case study surgery
 
Osce คณะ si 115
Osce คณะ si 115Osce คณะ si 115
Osce คณะ si 115
 
National test _2553_TU
National test _2553_TUNational test _2553_TU
National test _2553_TU
 
แนวทางเวชปฏิบัติในการรักษาผู้ป่วยมาลาเรียในประเทศไทย พ.ศ.2557
แนวทางเวชปฏิบัติในการรักษาผู้ป่วยมาลาเรียในประเทศไทย พ.ศ.2557แนวทางเวชปฏิบัติในการรักษาผู้ป่วยมาลาเรียในประเทศไทย พ.ศ.2557
แนวทางเวชปฏิบัติในการรักษาผู้ป่วยมาลาเรียในประเทศไทย พ.ศ.2557
 
กรณีศึกษาไต (Ns) แก้ไข
กรณีศึกษาไต (Ns)  แก้ไขกรณีศึกษาไต (Ns)  แก้ไข
กรณีศึกษาไต (Ns) แก้ไข
 

Viewers also liked

Clinical Practice Guideline of Acute Meningoencephalitis
Clinical Practice Guideline of Acute MeningoencephalitisClinical Practice Guideline of Acute Meningoencephalitis
Clinical Practice Guideline of Acute MeningoencephalitisUtai Sukviwatsirikul
 
ประชุมวิชาการ ศิริราช 52
ประชุมวิชาการ ศิริราช 52ประชุมวิชาการ ศิริราช 52
ประชุมวิชาการ ศิริราช 52vora kun
 
ประชุมวิชาการ ศิริราช 53
ประชุมวิชาการ ศิริราช 53ประชุมวิชาการ ศิริราช 53
ประชุมวิชาการ ศิริราช 53vora kun
 
Total parenteral nutrition
Total parenteral nutritionTotal parenteral nutrition
Total parenteral nutritionvora kun
 
Survivor NT step2 SIRIRAJ book 2
Survivor NT step2 SIRIRAJ book 2Survivor NT step2 SIRIRAJ book 2
Survivor NT step2 SIRIRAJ book 2vora kun
 
ศรว 51 ANS By Cmu
ศรว 51 ANS By Cmuศรว 51 ANS By Cmu
ศรว 51 ANS By Cmuvora kun
 
Finish giตัด
Finish giตัดFinish giตัด
Finish giตัดsaowaluk2556
 
Survivor NT step2 SIRIRAJ book 1
Survivor NT step2 SIRIRAJ book 1Survivor NT step2 SIRIRAJ book 1
Survivor NT step2 SIRIRAJ book 1vora kun
 
Abnormal pap smear ศิริราช ppt
Abnormal pap smear ศิริราช pptAbnormal pap smear ศิริราช ppt
Abnormal pap smear ศิริราช pptvora kun
 
ศรว 51 By Cmu
ศรว 51 By Cmuศรว 51 By Cmu
ศรว 51 By Cmuvora kun
 
Clinical medicine
Clinical medicineClinical medicine
Clinical medicineMoni Buvy
 
Survival for all draft 1 - 1
Survival for all draft 1  - 1Survival for all draft 1  - 1
Survival for all draft 1 - 1Domo Kwan
 
Survival for all draft 1 - 2
Survival for all draft 1  - 2Survival for all draft 1  - 2
Survival for all draft 1 - 2Domo Kwan
 
Survival for all draft 1 - 3
Survival for all draft 1  - 3Survival for all draft 1  - 3
Survival for all draft 1 - 3Domo Kwan
 
การอ่านค่า Ekg
การอ่านค่า Ekgการอ่านค่า Ekg
การอ่านค่า Ekgtechno UCH
 

Viewers also liked (16)

Clinical Practice Guideline of Acute Meningoencephalitis
Clinical Practice Guideline of Acute MeningoencephalitisClinical Practice Guideline of Acute Meningoencephalitis
Clinical Practice Guideline of Acute Meningoencephalitis
 
ประชุมวิชาการ ศิริราช 52
ประชุมวิชาการ ศิริราช 52ประชุมวิชาการ ศิริราช 52
ประชุมวิชาการ ศิริราช 52
 
ประชุมวิชาการ ศิริราช 53
ประชุมวิชาการ ศิริราช 53ประชุมวิชาการ ศิริราช 53
ประชุมวิชาการ ศิริราช 53
 
Total parenteral nutrition
Total parenteral nutritionTotal parenteral nutrition
Total parenteral nutrition
 
Survivor NT step2 SIRIRAJ book 2
Survivor NT step2 SIRIRAJ book 2Survivor NT step2 SIRIRAJ book 2
Survivor NT step2 SIRIRAJ book 2
 
ศรว 51 ANS By Cmu
ศรว 51 ANS By Cmuศรว 51 ANS By Cmu
ศรว 51 ANS By Cmu
 
EKG in ACLS
EKG in ACLSEKG in ACLS
EKG in ACLS
 
Finish giตัด
Finish giตัดFinish giตัด
Finish giตัด
 
Survivor NT step2 SIRIRAJ book 1
Survivor NT step2 SIRIRAJ book 1Survivor NT step2 SIRIRAJ book 1
Survivor NT step2 SIRIRAJ book 1
 
Abnormal pap smear ศิริราช ppt
Abnormal pap smear ศิริราช pptAbnormal pap smear ศิริราช ppt
Abnormal pap smear ศิริราช ppt
 
ศรว 51 By Cmu
ศรว 51 By Cmuศรว 51 By Cmu
ศรว 51 By Cmu
 
Clinical medicine
Clinical medicineClinical medicine
Clinical medicine
 
Survival for all draft 1 - 1
Survival for all draft 1  - 1Survival for all draft 1  - 1
Survival for all draft 1 - 1
 
Survival for all draft 1 - 2
Survival for all draft 1  - 2Survival for all draft 1  - 2
Survival for all draft 1 - 2
 
Survival for all draft 1 - 3
Survival for all draft 1  - 3Survival for all draft 1  - 3
Survival for all draft 1 - 3
 
การอ่านค่า Ekg
การอ่านค่า Ekgการอ่านค่า Ekg
การอ่านค่า Ekg
 

Similar to Nt2009 Complete Ans

Nle step 2_2009 si115-116 and nle_step_2_2009 nctms editors cut
Nle step 2_2009 si115-116 and nle_step_2_2009 nctms editors cutNle step 2_2009 si115-116 and nle_step_2_2009 nctms editors cut
Nle step 2_2009 si115-116 and nle_step_2_2009 nctms editors cutLoveis1able Khumpuangdee
 
Cvst 2
Cvst 2Cvst 2
Cvst 2med
 
Surgery of acquired heart disease อ.วรวงศ์ ศลิษฏ์อรรถกร
Surgery of acquired heart disease อ.วรวงศ์ ศลิษฏ์อรรถกรSurgery of acquired heart disease อ.วรวงศ์ ศลิษฏ์อรรถกร
Surgery of acquired heart disease อ.วรวงศ์ ศลิษฏ์อรรถกรpohgreen
 
การดูแลผู้ป่วยแน่นหน้าอก สงสัย Ischemiaxxx
การดูแลผู้ป่วยแน่นหน้าอก สงสัย  Ischemiaxxxการดูแลผู้ป่วยแน่นหน้าอก สงสัย  Ischemiaxxx
การดูแลผู้ป่วยแน่นหน้าอก สงสัย Ischemiaxxxeremslad
 
Motor weakness and Cerebrovascular Disease
Motor weakness and Cerebrovascular DiseaseMotor weakness and Cerebrovascular Disease
Motor weakness and Cerebrovascular DiseaseNarongrit Kasemsap
 
Interesting case ACLS.pdf
Interesting case ACLS.pdfInteresting case ACLS.pdf
Interesting case ACLS.pdfStafarne
 
Clinical practice guidelines mild head injury
Clinical practice guidelines mild head injuryClinical practice guidelines mild head injury
Clinical practice guidelines mild head injurySiwaporn Khureerung
 
Slide โรคเรื้อรัง สปสช
Slide โรคเรื้อรัง สปสชSlide โรคเรื้อรัง สปสช
Slide โรคเรื้อรัง สปสชคิดดี ทำดี
 
CPG Thai Stroke infarct retrieved since 2555
CPG Thai Stroke infarct retrieved since 2555CPG Thai Stroke infarct retrieved since 2555
CPG Thai Stroke infarct retrieved since 2555Thorsang Chayovan
 
Ortho conferences
Ortho conferencesOrtho conferences
Ortho conferencesyuricommand
 

Similar to Nt2009 Complete Ans (20)

Nle step 2_2009 si115-116 and nle_step_2_2009 nctms editors cut
Nle step 2_2009 si115-116 and nle_step_2_2009 nctms editors cutNle step 2_2009 si115-116 and nle_step_2_2009 nctms editors cut
Nle step 2_2009 si115-116 and nle_step_2_2009 nctms editors cut
 
Blunt chest injury
Blunt chest injuryBlunt chest injury
Blunt chest injury
 
Spinal injury
Spinal injurySpinal injury
Spinal injury
 
Cvst 2
Cvst 2Cvst 2
Cvst 2
 
Surgery of acquired heart disease อ.วรวงศ์ ศลิษฏ์อรรถกร
Surgery of acquired heart disease อ.วรวงศ์ ศลิษฏ์อรรถกรSurgery of acquired heart disease อ.วรวงศ์ ศลิษฏ์อรรถกร
Surgery of acquired heart disease อ.วรวงศ์ ศลิษฏ์อรรถกร
 
Acute coronary syndrome 2010
Acute coronary syndrome 2010Acute coronary syndrome 2010
Acute coronary syndrome 2010
 
Septic abortion
Septic abortionSeptic abortion
Septic abortion
 
การดูแลผู้ป่วยแน่นหน้าอก สงสัย Ischemiaxxx
การดูแลผู้ป่วยแน่นหน้าอก สงสัย  Ischemiaxxxการดูแลผู้ป่วยแน่นหน้าอก สงสัย  Ischemiaxxx
การดูแลผู้ป่วยแน่นหน้าอก สงสัย Ischemiaxxx
 
Motor weakness and Cerebrovascular Disease
Motor weakness and Cerebrovascular DiseaseMotor weakness and Cerebrovascular Disease
Motor weakness and Cerebrovascular Disease
 
Cpg ped
Cpg pedCpg ped
Cpg ped
 
Interesting case ACLS.pdf
Interesting case ACLS.pdfInteresting case ACLS.pdf
Interesting case ACLS.pdf
 
Nl
NlNl
Nl
 
Clinical practice guidelines mild head injury
Clinical practice guidelines mild head injuryClinical practice guidelines mild head injury
Clinical practice guidelines mild head injury
 
Rheumatoid arthritis
Rheumatoid arthritisRheumatoid arthritis
Rheumatoid arthritis
 
Interesting case
Interesting caseInteresting case
Interesting case
 
Ihd
IhdIhd
Ihd
 
Slide โรคเรื้อรัง สปสช
Slide โรคเรื้อรัง สปสชSlide โรคเรื้อรัง สปสช
Slide โรคเรื้อรัง สปสช
 
Ortho case
Ortho caseOrtho case
Ortho case
 
CPG Thai Stroke infarct retrieved since 2555
CPG Thai Stroke infarct retrieved since 2555CPG Thai Stroke infarct retrieved since 2555
CPG Thai Stroke infarct retrieved since 2555
 
Ortho conferences
Ortho conferencesOrtho conferences
Ortho conferences
 

More from vora kun

CPR 2010 อ ปริญญา รามา
CPR 2010 อ ปริญญา รามาCPR 2010 อ ปริญญา รามา
CPR 2010 อ ปริญญา รามาvora kun
 
NT step2 march 53
NT step2 march 53NT step2 march 53
NT step2 march 53vora kun
 
Thai Osteoporosis guideline 2553
Thai Osteoporosis guideline 2553Thai Osteoporosis guideline 2553
Thai Osteoporosis guideline 2553vora kun
 
ortho 02 orthopaedic complication & prevention + orthopaedic trauma (practica...
ortho 02 orthopaedic complication & prevention + orthopaedic trauma (practica...ortho 02 orthopaedic complication & prevention + orthopaedic trauma (practica...
ortho 02 orthopaedic complication & prevention + orthopaedic trauma (practica...vora kun
 
ortho 06 common ortho dis 2 edited 12 mar 10
ortho 06 common ortho dis 2 edited 12 mar 10ortho 06 common ortho dis 2 edited 12 mar 10
ortho 06 common ortho dis 2 edited 12 mar 10vora kun
 
ortho 05 common rheumatic dx rx
ortho 05 common rheumatic dx rxortho 05 common rheumatic dx rx
ortho 05 common rheumatic dx rxvora kun
 
ortho 01 management of open fracture-update by kk 31052010
ortho 01 management of open fracture-update by kk 31052010ortho 01 management of open fracture-update by kk 31052010
ortho 01 management of open fracture-update by kk 31052010vora kun
 
ortho 04 drugs in orthopaedic (principle & common use)
ortho 04 drugs in orthopaedic (principle & common use)ortho 04 drugs in orthopaedic (principle & common use)
ortho 04 drugs in orthopaedic (principle & common use)vora kun
 
ortho 03 principle of closed reduction in fracture and dislocation
ortho 03 principle of closed reduction in fracture and dislocationortho 03 principle of closed reduction in fracture and dislocation
ortho 03 principle of closed reduction in fracture and dislocationvora kun
 
ortho 02 orthopaedic complication & prevention + orthopaedic trauma (practica...
ortho 02 orthopaedic complication & prevention + orthopaedic trauma (practica...ortho 02 orthopaedic complication & prevention + orthopaedic trauma (practica...
ortho 02 orthopaedic complication & prevention + orthopaedic trauma (practica...vora kun
 
SWU CXR interpretation
SWU  CXR interpretationSWU  CXR interpretation
SWU CXR interpretationvora kun
 
Step3 Tutorial by SWU book2
Step3 Tutorial by SWU book2Step3 Tutorial by SWU book2
Step3 Tutorial by SWU book2vora kun
 
NeuroSx step2 Review
NeuroSx step2 ReviewNeuroSx step2 Review
NeuroSx step2 Reviewvora kun
 

More from vora kun (13)

CPR 2010 อ ปริญญา รามา
CPR 2010 อ ปริญญา รามาCPR 2010 อ ปริญญา รามา
CPR 2010 อ ปริญญา รามา
 
NT step2 march 53
NT step2 march 53NT step2 march 53
NT step2 march 53
 
Thai Osteoporosis guideline 2553
Thai Osteoporosis guideline 2553Thai Osteoporosis guideline 2553
Thai Osteoporosis guideline 2553
 
ortho 02 orthopaedic complication & prevention + orthopaedic trauma (practica...
ortho 02 orthopaedic complication & prevention + orthopaedic trauma (practica...ortho 02 orthopaedic complication & prevention + orthopaedic trauma (practica...
ortho 02 orthopaedic complication & prevention + orthopaedic trauma (practica...
 
ortho 06 common ortho dis 2 edited 12 mar 10
ortho 06 common ortho dis 2 edited 12 mar 10ortho 06 common ortho dis 2 edited 12 mar 10
ortho 06 common ortho dis 2 edited 12 mar 10
 
ortho 05 common rheumatic dx rx
ortho 05 common rheumatic dx rxortho 05 common rheumatic dx rx
ortho 05 common rheumatic dx rx
 
ortho 01 management of open fracture-update by kk 31052010
ortho 01 management of open fracture-update by kk 31052010ortho 01 management of open fracture-update by kk 31052010
ortho 01 management of open fracture-update by kk 31052010
 
ortho 04 drugs in orthopaedic (principle & common use)
ortho 04 drugs in orthopaedic (principle & common use)ortho 04 drugs in orthopaedic (principle & common use)
ortho 04 drugs in orthopaedic (principle & common use)
 
ortho 03 principle of closed reduction in fracture and dislocation
ortho 03 principle of closed reduction in fracture and dislocationortho 03 principle of closed reduction in fracture and dislocation
ortho 03 principle of closed reduction in fracture and dislocation
 
ortho 02 orthopaedic complication & prevention + orthopaedic trauma (practica...
ortho 02 orthopaedic complication & prevention + orthopaedic trauma (practica...ortho 02 orthopaedic complication & prevention + orthopaedic trauma (practica...
ortho 02 orthopaedic complication & prevention + orthopaedic trauma (practica...
 
SWU CXR interpretation
SWU  CXR interpretationSWU  CXR interpretation
SWU CXR interpretation
 
Step3 Tutorial by SWU book2
Step3 Tutorial by SWU book2Step3 Tutorial by SWU book2
Step3 Tutorial by SWU book2
 
NeuroSx step2 Review
NeuroSx step2 ReviewNeuroSx step2 Review
NeuroSx step2 Review
 

Nt2009 Complete Ans

  • 1. ชุดที่ 1 Medicine 1.) ตอบ 4. Defibillation Ventricular fibrillation - EKG : no p wave ,no QRS cpx can be identified, irregular rhythm ,rate >300 RA J Management : VF and pulseless : Defibillation เพิ่มเติม VF และคลํา pulseได RI 1.vital sign stable : amiodarone SI 2.vital sign unstable(shock) : cardioversion 2.) ตอบ 1. 3rd heart block AJ 3rd heart block พบวา P กับ QRS จะไมสัมพันธกัน และจะมีอาการไดบอย, 2nd IR mobitz I พบวา PR ยาวขึ้นเรื่อยๆ จน QRS หายไปเปนบางครั้งแลวจึงกลับมาเปนปกติ ไม IR คอยมีอาการ, 2nd mobitz II พบวา PR ยาวเทาๆเดิม แตอยูๆ QRS ก็หายไป พบอาการได บาง, 1st degree พบ PR ยาวขึ้นเฉยๆ แลวก็ไมมีอาการ S 3.) ตอบ 4. adrenaline im AJ Anaphylaxis criteria dx มี 3วิธี IR 1.มีอาการทางskin + Rs dysfn or BP drop IR 2.มี Hx สัมผัสสารที่นาจะเปน allergen + อาการ 2/4 (1.skin or mucosa 2.RS dysfn 3. BP drop 4.GI) S 3.มี Hx สัมผัส known allergen + SBP < 90 หรือ ลดลงจาก30% baseline Management : adrenaline im 1:1000 dose 0.01 ml/kg ( child max 0.3ml, adult 0.5ml) ใหซ้ําได 1-2 ครั้ง หางกัน 10-15 นาที 4.) ตอบ 5. Paroxysmal nocturnal hemoglobinuria(PNH) PBS : NCNC / เปน intravascular hemolysis (ปสสาวะมีสีชาหรือโคลา) , อาการ แสดง : pancytopenia
  • 2. การวินิจฉัยจะตองทํา Ham’s test (acid serum test) หรือ sucrose hemolysis test 7.) ตอบ 2. Prostigmine test choice ขออื่น AIHA , HS ,Thalassemia เปน extravascular hemolysis เนื่องจากนึกถึง Myasthenia gravis / Hx&PE : fluctuating weakness increased by G-6-PD เปน intravascular hemolysis / PBS : bite cell ,Heinz bodies exertion, Weakness increases during the day and improves with rest, proximal weakness มักเปนผูชาย(XR) และมักมีประวัติ drug-induced greater than distal muscle weakness, ptosis, limited to the EOM J 5.) ตอบ 3. AIHA Ix : NEOSTIGMINE TEST ( Prostigmin test) 1-2 mg im evaluate 20-40 mins RA จาก ญ วัยรุน ซีด เหลืองเล็กนอย ตับไมโต มามโตเล็กนอย reti ขึ้น (แตทําไมMVC Rx : Anticholinesterase medication (oral pyridostigmine) เปน symptomatic RI เล็ก ซึ่งจะนึกถึง thalassemia, iron def. หากจะตอบ thalassemia ตองเปน H disease ที่ treatment; onset 15-30 mins, last for 3-4 hrs; begin with moderate dose 60mg 3-5 times SI สามารถมี reti ขึ้นไดประมาณ 5 %) per day, maximal dose 120mg every 3 hrs during daytime; adverse effect : increased Thalassemia ไมมีอาการ จนถึง ซีดเหลืองรุนแรง ตับมามโต chipmuck face muscle weakness and muscarinic effect, antidote is propantheline bromide AJ hyperpigmentation MCV เล็ก 8.) ตอบ 1. ขอความยินยอมพอแมเพื่อ LP IR Iron def ซีด เหนื่อย มีประวัติเสียเลือดเรื้อรัง glossitis, koilonychias, angular ผูปวยเด็กอายุ 8 เดือน มีอาการไขสูง มี Brudzinski’s sign positive จากโจทยทําใหนึก stomatitis,Wbc,plt ปกติ พบ serum ferritin, Serum Iron, Transferrin Saturation ต่ํา TIBC ถึงภาวะ meningitis มากที่สุดผูปวยรายนี้ อาการที่พบไดบอยของภาวะนี้คือ ไข ปวดศีรษะ IR สูง MVC เล็ก ซึม สับสน คลื่นไสอาเจียน ปวดคอ คอแข็ง ตรวจรางกายอาจพบ ระดับความรูสึกตัวที่ S G6PD มักพบใน ช> ญ ไมพบมามโต ซีดฉับพลันและเปนชั่วคราว อาจเหลือง ลดลง และอาการแสดงของ Meningeal irritation ทั้ง Kernig’s และ Brudzinski’s sign อาจ AJ เล็กนอย อาจมีปสสาวะดํา พบ Heinz body, ghost cell มี nuchal rigidity นอกจากนี้ยังอาจพบความผิดปกติของ CN รวมดวย เชน III ,IV, VIII / HS มีอาการรุนแรงเมื่อมีภาวะติดเชื้อหรือมีไข มาพบแพทยดวย เหลือง มักพบมามโต IR การวินิจฉัย gold standard ในผูปวยที่สงสัยภาวะนี้ตองทํา Lumbar puncture เพื่อดู CSF มักพบภาวะแทรกซอนเปนนิ่วในถุงน้ําดี พบ spherocyte, reticulocyte มากขึ้น profile วาเปนmeningitis หรือไม และเกิดจากเชื้อกลุมใด อยางไรก็ตาม มีขอหามในการทํา IR AIHA warm type – ญ>ช อายุ 20-40 ตรวจพบโลหิตจาง เหลืองเล็กนอย อาจจบมาม LP ถาผูปวยมี Focal neurologic sign,มี Evidence of space-occupying lesion with S หรือตับโตเล็กนอย reticulocyte มากขึ้น wbc ต่ํา หรือ สูงขึ้น plt ปกติ increased ICP , Immunocompromised host, , มี new onset of seizure ภายใน 1 สัปดาห 6.) ตอบ 5. FFP และมี papilledema ผูที่มีขอหามขอใดขอหนึ่ง ควรไดรับการทํา CT brain กอน เนื่องจาก I/C of FFP - 1.replace of multiple coagulation factors ex. liver disease , DIC , กลัว brain herniation ถาพบรอยโรคใน brain จริง จะไมทํา LP แลว treat ไปเลย massive transfusion(Pt,APTT ratio>1.5x normal) การรักษาคือ การให Empirical antibiotics ทันทีหลังจากการทํา LP และ Hemoculture 2.Treatment of TTP, HUS แลว นอกจากนี้การให dexamethasone ยังมีประโยชนในผูปวยที่เปน pneumococcal meningitis
  • 3. กลับมาพูดถึงผูปวยรายนี้ มีปญหาคือมี anterior fontanelle โปง เขาใจวานาจะหมายถึง including sleepiness and impulse control disorders, these side effects resolve upon มี ICP สูง เราไมแนใจวาเทาที่โจทยใหมาเปน C/I ในการทํา LP หรือไม แตคิดวาไมนาจะ lowering the dose or discontinuing the medication หามทํา LP เพราะเด็กยังไมมีอาการ focal neurologic sign ที่ชัดเจน (มั้ง?) ดังนั้นขอตอบขอ - For patients younger than 65 years, use a dopamine agonist and then add 1 คือขอความยินยอมจากพอแมเพื่อ LP (แตไมแนใจวาตอง CT กอนรึปาวนะ แตหลักการ levodopa/PDI when the dopamine agonist (with or without an MAO-B inhibitor) no J ตามนี้ละ) longer provides good control of motor symptoms. Dopamine agonists may provide good RA 9.) ตอบ 5. ให amitriptyline symptom control for several years. RI อาการปวดศีรษะแบบตุบๆขางเดียว เปนๆหายๆ คิดถึง Migraine without aura ซึ่ง - For patients who are demented or older than 70 years (those who may be prone to SI หลักการรักษาคือ 1. หลีกเลี่ยงสิ่งกระตุน & 2. Medication โดย adverse effects, such as hallucinations, from dopamine agonists), and for those likely to Acute : paracetamol, NSAIDs; Ergotamine tartrate; Triptans ( 5HT agonists) require treatment for only a few years, not to use a dopamine agonist and depend on AJ Chronic : ใหยา : β blocker, CCB, TCA, SSRIs เพื่อปองกันเมื่อ levodopa/PDI as primary symptomatic therapy IR 1. ปวดบอยมากกวา 2 ครั้งตอเดือน ซึ่งทําใหเกิด disability มากกวา 3 วัน - benhexal(Anticholinergic) medications provide good tremor relief in 2. ปวดแตละครั้งรุนแรงมาก approximately 50% of patients but do not improve bradykinesia or rigidity IR 3. ปวดมากกวา 2 ครั้งตอสัปดาห - Selegiline (MAO-B inhibitors) provide mild symptomatic benefit, have excellent S 4. ไมสามารถใชยา acute medication ได side effect profiles, and may improve long-term outcomes, a good choice as initial AJ 5. ใชยา acute medication มากเกินไป treatment for many patients, delays the need for levodopa therapy in early Parkinson 10.) ตอบ 2. levodopa IR disease Parkinson disease is a progressive neurodegenerative disorder associated with a 11.) ตอบ 1. Phenytoin IR loss of dopaminergic neurons in the substantia nigra / Cardinal signs : resting tremor, คิดถึงภาวะ generalized tonic-clonic seizures ยาหลักในการรักษาก็คือ Phenytoin S cogwheel rigidity, bradykinesia, Postural instability Phenobarbital Carbamazepine สวน diazepam นั้นมักเลือกใชเปนยาชนิดแรกในกรณี - levodopa: standard of symptomatic treatment for Parkinson disease, fewest status epilepticus adverse effects in the short term, its chronic use is associated with the development of 12.) ตอบ 1. Hypokalemia fluctuations and dyskinesias. แขนขาออนแรง ตองคิดถึง hypokalemia ซึ่งรายนี้มีประวัติได HCTZ ซึ่งทําให - Bromocriptine(Dopamine agonists): provide moderate symptomatic benefit and hypokalemia ได rarely cause fluctuations and dyskinesias , but more side effects than levodopa,
  • 4. 13.) ตอบ 4. Ceftriaxone ชวงอายุประมาณ 30-40 yrs อาจพบ IgA nepropathy, postinfectious, lupus ..... / การรักษา เนื่องจากผูปวยมีภาวะ urosepsis กลาวคือมีอาการไขสูง หนาวสั่น ปสสาวะแสบขัด ใหเปน ACEI or ARB , steroid ผล U/A พบ WBC 50-60 จึงเขาไดกับภาวะ UTI รวมกับมี sign of SIRS > 2ใน 4 ขอ ไดแก 17.) ตอบ 5. Primary polydipsia 1. T > 38 or < 36 C 2. RR > 24 3. P > 90 4. WBC > 12000 or < 4000 or band form > 10 ขอนี้บอกตามตรงวา ขอสอบมันงงๆนะ คือถาโจทยจําตัวเลขมาถูกแลว ก็ตองมาแปล J % จึงควรใหการรักษาดวย IV quinolone / ampicillin + gentamicin / ceftriaxone ผล lab กันนะ คนนี้มี hyponatremia แลว urine Na ต่ํา (ปกติ 15-250 mEq/L/day) และ RA 14.) ตอบ 1. urineVMA urine osmol. ต่ํา (ปกติ 500-800 mOsm/kg) มาตัด choice กัน RI Pheochromocytoma SIADH คือ รางกายหลั่ง ADH ซึ่งทําหนาที่ดูดกลับน้ํา มากเกินไป ก็ทําให SI อาการและอาการแสดง 1. Headache 2.Diaphoresis 3.Palpitations 4.Severe hyponatremia ได แต urine ก็จะตอง เขมขนสิ ควรมี urine osmol และ urine Na สูง จึงไม hypertension (These 4 characteristics together are strongly suggestive of a ตอบขอนี้ AJ pheochromocytoma) 5.Tremor 6.Nausea 7.Weakness 8.Anxiety 9.Epigastric pain Low salt intake กินเกลือนอยไมใชสาเหตุที่ทําให hyponatremia ไดมากขนาดนี้ IR 10.Flank pain 11.Constipation 12.weight loss Renal wasting คือ เสีย Na ไปทางไต ก็ควรมี urine Na สูงสินะ Dx 1.Plasma metanephrine testing has the highest sensitivity (96%) for detecting a Thiazide มี side effect ทําให hyponatremia ไดจากการที่ขับ Na ทางปสสาวะเพิ่มขึ้น IR pheochromocytoma, lower specificity (85%). ก็นาจะมี urine Na สูงดวย S 2.24-hour urinary collection for catecholamines ,vanillylmandelic acid and Primary polydipsia เขาไดมากสุด เพราะทําใหมี hyponatremia , urine Na ต่ํา และ AJ metanephrines has a sensitivity of 87.5% and a specificity of 99.7% urine osmol. ต่ํา ; แตก็ไมเขาใจวาเกี่ยวอะไรกับประวัติ CA lung ตอนแรกคิดวาจะให 3.CT , MRI abdomen คิดถึง SIADH ที่พบไดใน Small cell lung cancer แต lab ไมเขาอะ IR 15.) ตอบ 3. Serum electrolyte 18.) ตอบ 1. Calcium gluconate IR ในผูปวยรายนี้จากประวัติและการตรวจรางกายทําใหนึกถึงภาวะ primary เนื่องจากมีภาวะ hyperkalemia (K =6.0)ทําใหเกิด cardiac arrhythmia ได ; อาการ S hyperaldosteronism เนื่องจากมี muscle weakness จาก potassium ต่ํา ไมมีอาการใจสั่น ไดแก Generalized fatigue,Weakness ,Paresthesias ,Paralysis ,Palpitations หรืออาจ เหงื่อ ออกหนาแดง อันจะพบไดใน pheochromocytoma ดังนั้นการสง Ix ที่เหมาะสมที่สุด asymptomatic / Risk ไดแก Acute or chronic renal failure, Trauma including crush ในผูปวยรายนี้ นาจะเปน serum electrolyte มากที่สุด injuries (rhabdomyolysis) or burns ,Ingestion of foods high in potassium (eg, bananas, 16.) ตอบ 4. Prednisolone oranges, high-protein diets, tomatoes, salt substitutes) ,ยา - Potassium supplements, ผูปวยมี nephrotic nephritic symdrome คิดถึง glomerular disease : primary or potassium-sparing diuretics, NSAIDs, beta-blockers, digoxin, succinylcholine, and secondary จากโจทยไมไดบอก clue ที่สงสัยวามีโรคประจําตัวใดๆ / คิดถึง primary กอน digitalis glycoside ,Redistribution - Metabolic acidosis (diabetic ketoacidosis [DKA]),
  • 5. catabolic state Rx : สงตรวจEKG,หยุดpotassium sparing drug, ให Calcium chloride or HRCT ; CXR : Cystic shadow, bronchial wall thickening (Tramline and ring shadows); calcium gluconate (Kalcinate) Treatment : postural drainage, ATB, Bronchodilator (ถามี asthma, COPD รวมดวย) Adult 21.) ตอบ 1. Sputum AFB - Calcium chloride: 5 mL of 10% sol IV over 2 min (stop infusion if bradycardia ) เนื่องจาก อาการที่เขาไดกับ pulmonary TB คือ ไขตอนเย็นๆ ไอมีเสมหะ 3 wk ตรวจ J - Calcium gluconate: 10 mL of 10% sol IV over 2 min (stop infusion if bradycardia ) film พบ alveolar infiltration และ Cavity ที่ RUL (suggest actibe TB) การตรวจพบ AFB RA Pediatric positive ก็จะชวยยืนยันผลได (แตถาตรวจไมพบก็คง treat as TB อยูดี?) Regimen ที่ควร RI - Calcium chloride: 0.2 mL/kg/dose of 10% sol IV over 5 min; not to exceed 5 mL (stop ไดรับในการรักษาคือ 2 IRZE + 4 IR [Co-amoxiclav และ levofloxacin จริงๆแลวก็ SI infusion if bradycardia ) สามารถใชในการรักษา TB ไดเชนกัน โดยใชรักษาในกลุม multidrug resistance TB ซึ่ง - Calcium gluconate: 100 mg/kg (1 mL/kg) of 10% sol IV over 3-5 min; not to exceed 10 ตองใหรวมกับยาอื่นๆอีก คือ Clofazimene Moxifloxacin และ Amikacin] AJ mL (stop infusion if bradycardia ) ในผูปวย HIV จะมีโอกาสตรวจพบเชื้อจากเสมหะไดนอย ดังนั้นในผูปวย HIV เพื่อ IR 19.) ตอบ 1. Hypo K การวินิจฉัยอาจตรวจโดย bronchoscopy, tissue biopsy, blood culture ก็ขอนี้ ถาม S-E ของ HCTZนะ ไดแก 22.) ตอบ 2. วินิจฉัยใหไดวาเปนมะเร็งจริง IR Hypo “ขนม” K Na Mg แลวก แตสําหรับโจทยขอนี้ คิดวา เนื่องจากแคสงสัยวาเปนมะเร็งปอดกอนอื่นก็ตองวินิจฉัย S Hyper Ca glycemia lipidemia uricemia ใหไดวาเปนมะเร็งปอดจริง จึงคิดวานาจะตอบขอ 2 นะ ขางลางนี้เปนความรูที่ไปหามาให AJ สําหรับอาการแขนขาออนแรง เนี่ยกcommon จาก Hypo K แหละ ไมเกี่ยวกับการตอบขอนี้เลย Hypo Na N/V fatique ซึม ชัก โรคมะเร็งปอด : มีสาเหตุจากสารเคมีอนุภาคหลายชนิดไดแก แอสเบสตอส เรดอน IR Hypo Ca Neuromuscular irritability คลอโรเมธินอีเธอร โพลีอะโรมาติก ไฮโดรคารบอน อารเซนิก โครเมียม นิกเกิล อยางไรก็ IR HypoMg N/V CNS suppression hyporeflexia (คลายๆ preeclampsiaที่ตอง ใหMg ตามจากประวัติการสัมผัสทําใหสามารถวินิจฉัยวาเกิดจากการทํางาน ประวัติที่ทําให S แกเรยเนอะ ^^) วินิจฉัยยากคือ ประวัติการสูบบุหรี่ สวนใหญเมื่อวินิจฉัยกอนจะโตมากแลว และผูปวยจะ 20.) ตอบ 2. Bronchiectasis ถึงแกกรรมในเวลาไมนาน อาการคือมีอาการของการระคายเคืองของเยื่อหุมปอด เนื่องจากไอเรื้อรังมานาน มีเสมหะเขียว และ peribronchial thickening ** หลอดลม ถุงลมในปอด ทําใหมีอาการหอบเหนื่อย ไอเปนเลือด นอกจากนี้มะเร็งปอดยัง Bronchiectasis เปน chronic infection ทําใหbronchus+bronchiole dilate อยางถาวร (เปน สามารถแพรกระจายไปยังอวัยวะอื่นทําใหมีอาการของอวัยวะนั้นๆ การสูบบุหรี่ทําใหเปน Obstructive lung) , เกิดจากเชื้อ : H. influenzae, S. pneumo, S. aureus, P. aeruginosa; มะเร็งปอดไดและมีโอกาสที่จะเปนจากบุหรี่มากกวาการประกอบอาชีพหลายเทา อาการ : ไอเรื้อรัง, เสมหะเขียวปริมาณมาก (อาจมีเลือดปน); Ix: CXR, Sputum culture,
  • 6. 23.) ตอบ 4. Bronchogenic carcinoma Complicated pleural effusion มีลักษณะดังนี้ อาการแนนหนาอก เหนื่อย หนาและแขนบวม มี face and upper extremity edema , 1.Frank pus or empyema thoracis superficial vein dilatation at chest wall เขาไดกับ SVC (ในระยะแรกอาจจะ ไมแสดง 2.มีorganisim (พบในผูปวยรายนี้) อาการใด ๆ โดยเฉพาะเปน partial obstruction แตโดยสวนใหญจะมีอาการ และอาการ 3.pH<7.2 J แสดงเล็ก ๆ นอย ๆ ใหเห็นบาง แตหาก total obstruction จะแสดงอาการตางๆโดย 4.Sugar<60mg/dl (พบในผูปวยรายนี้) RA Dyspnea เปนอาการที่พบมากที่สุด อาการอื่นๆ ประกอบดวย facial swelling, head 5.Loculate parapneumonic effusion RI fullness, cough, arm swelling, chest pain, dysphagia, orthopnea, distorted vision, Transudate Exudate Empyema I hoarseness, stridor, headache, nasal stuffiness, nausea, pleural effusions, and light- thoracis S headedness). pH 7.4-7.6 <7.4 <7.2 J สาเหตุของ SVC obstruction Cell <1,000 1,000-5,000 >5,000 PMN RA มากกวา 80% เกิดจาก malignant mediastinal tumors : Bronchogenic carcinomas L,Mo PMN RI พบ 75-80% ในภาวะนี้ โดยสวนใหญเปน small-cell carcinomas ; Non-Hodgkin Specific gravity <0.015 >0.015 SI lymphoma (โดยเฉพาะ large cell type) พบประมาณ 10-15% ; Malignant อื่นๆที่พบนอย LDH <200 200-1,000 >1,000 ประกอบดวย Hodgkin disease, metastatic cancers, primary leiomyosarcomas of the LDH ratio <0.6 >0.6 AJ mediastinal vessels, and plasmocytomas. Protein ratio <0.5 >0.5 Nonmalignant อื่นๆที่เปนสาเหตุของ SVC obstruction ประกอบดวย mediastinal IR Serum pleural-albumin >1.2 <1.2 fibrosis; vascular diseases เชน aortic aneurysm, vasculitis, and arteriovenous fistulas ; gradient (serum albumin- IR infections such as histoplasmosis, tuberculosis, syphilis, and actinomycosis ; benign pleural albumin) S mediastinal tumors เชน teratoma, cystic hygroma, thymoma, and dermoid cyst ; cardiac 25.) ตอบ 3. Organophosphate causes, เชน pericarditis and atrial myxoma; and thrombosis related to the presence of Organophosphate กระตุน Parasym นะ อาการกคือ SLUDGE 3B central vein catheters. S Sweating + Salivation 24.) ตอบ 4. ใหยาเดิม + ใส ICD L Lacrimation ขอนี้คิดวาเปนaspiration pneumonia เชื้อที่พบนาจะเปนพวก anaerobe จาก GI tract U Urination การใหยา co-amoxyclav ก็สามารถคลุมเชื้อได แตควรใสICDเพื่อระบาย pleural effusion D Diarrhea ดวยเพราะเปน complicated pleural effusion (เปน indicationใหใสICD) G GI ปวดทอง ( จาก motility เยอะมั้ง)
  • 7. E Emesis 31.) ตอบ 2. มีโอกาสเกิด embolic stroke มากกวาคนทั่วไป 15-20% B Bradycardia Diactolic rumbling murmur นึกถึง MS ซึ่งมักเกิดจาก Rheumatic heart disease B Bronchorhea ( secretion เยอะ) หลังจาก Rheumatic fever / MS ทําใหมีความเสี่ยงตอ embolic stroke มากขึ้น/ ขอ3ผิด B Bronchospasm เพราะ MS ทําใหเกิด pulmonary hypertension ไดจริง แตเนื่องจากลิ้นหัวใจตีบ ไมใชรั่ว J + miosis + Nicotinic effect เชน M.fasiculation Resp M paralysis + 32.) ตอบ 3. Bacterial endocarditis RA CNS effect เชน seizure Coma CNS depression ขอนี้ผูปวยมาดวย ไข + HF จึงสงสัย bacterial endocarditis โดยวินิจฉัยตาม Duke RI 26.) ตอบ 1. Botulinum toxin criteria (ลองไปหาดูเองนะ) ซึ่งในรายนี้ มี valvular regurgitation, conjuctival hemorrhage, SI กตอบขอนี้แหละมีอยูขอเดียว อาการเปนไดตั้งแต หายไดเอง จนถึง เสียชีวิต อาการคือ fever, Osler’s node(ตุมดําแดงกดเจ็บที่ปลายนิ้ว) ครบ 1 major + 3 minor criteria symmetrical descending paralysis ,abd cramp ,N/V ,diarrhea ,Resp failure การรักษาคือ atrial myxoma อาจจะไมมีอาการเลย หรือ มาดวยอาการจาก mass effect หรือ จากการ AJ admit observe cli , +-gastric lavage ที่ tumor ไป involve muscle ปกติ ทําใหเกิด heart failure IR 27.) คือ ขอนี้ มันเปนลักษณะ typical ของ Arsenic poisoning เรยแหละ แลว antidote กคือ Acute rheumatoid carditis (= acute rheumatic fever) มีอาการไดดังตอไปนี้ carditis, Dimercaprol (BAL in Oil) หรือ Succimer (DMSA) หรือ Dimerval (DMPS) ไมเห็นมีตํา polyarthritis, chorea, erythema marginatum, and subcutaneous nodules เปนตน IR ตอบเรยอะ งง Rupture chordae tendinae ก็จะมีอาการของ mitral valve regurgitation, acute left- S 29.) ตอบ 1. Acute thyroiditis sided CHF, pulmonary edema, tachypnoea and coughing เปนตน AJ ผูปวยรายนี้เขาไดกับ Thyrotoxicosis (ภาวะที่รางกายมี thyroid h. มากกวาปกติ) มี 33.) ตอบ 2. Epinephrine อาการเหนื่อยงาย, ใจสั่น, sweating, tremor, diarrhea, กินเยอะแตน้ําหนักลด; PE : fine Pulseless Arrest Algorithm : ผูปวยไมมี pulse แตมี electrical activity จึงจัดอยูใน IR tremor (hand), lid lag & lid retraction, onycholysis PEA ขั้นตอไปจึง ให epinephrine 1 mg. IV/IO repeat every 3-5 min IR พิจารณาลักษณะของกอน เปนได 3 แบบ คือ 1. Diffuse goiter ; 2. Solitary nodule ; 3. 35.) ตอบ 3. Eisenmenger Syndrome S Multinodular goiter รายนี้เขาไดกับ diffused goiter โดย ซึ่งเปนภาวะที่พบไดในผูปวยที่มี large congenital cardiac หรือ surgically created 1. Grave’s dis. : thyrotoxicosis, exopthalmos, pretibial myxedema, คลําได thrill extracardiac left-to-right shunts เปนผลใหเกิดincreased pulmonary blood flowตามมา. 2. Thyroiditis : แบงเปน acute, subacute, chronic (depend on duration and course of ตรสจรางกายสามารถพลลักษณะตางๆไดดังนี้ Central cyanosis (differential cyanosis in disease) คิดวานาจะตอบ acute thyroiditis เพราะมี pain, neck swelling, fever (สวน the case of a PDA) , Clubbing , Jugular venous pulse (wave may be A-wave dominant, suppurative thyroiditis นาจะ Unilateral neck pain และจะบวมแดงกดเจ็บบริเวณนั้นดวย) and, in the presence of a significant tricuspid regurgitation, the V wave may be 3. อื่นๆ เชน thyrotoxicosis factitious prominent; central venous pressure may be elevated) , Precordial palpation reveals a กรณีที่ประวัติแยกไดไมชัดเจนจะสงตรวจ I131 ดูการ uptake เพื่อแยกโรคตอไป right ventricular heave and, frequently, a palpable S2. , Loud P2 , High-pitched early
  • 8. diastolic murmur of pulmonic insufficiency , Right-sided fourth heart sound , Pulmonary 38.) ตอบ 5. U/S whole abdomen ejection click , Single S2 คิดวาเปน ruptured AAA with decompensated shock Ix ทํา U/S เพื่อ diagnosis and 36.) EKG F/U CT if plan surgery แตถา shock อยูหามทํา ดังนั้น จึงสง U/S whole abdomen 39.) ตอบ 5. NSS J ผูปวย shock (tachycardia, BP drop) แบบ hypovolemic (flat neck v.) ตอง resuscitate RA Sinus arrest ดวย isotonic solution คือ NSS RI 40.) ตอบ 4. Stool occult blood (จากชีทอ.กมล recommendation for Thai adults) SI Sinus bradycardia - grade ก.(ควรทํา) แนะนําตรวจ stool occult blood/colonoscopy ทุก 5 ปแนะนําใน ทุกอายุ ถามีประวัติ family history of colorectal CA, colonic polyp AJ - grade ข. (นาทํา) แนะนําตรวจ stool occult blood ทุก 5 ป ที่อายุ> 40 ป จากที่คนนี้ IR ไมมีประวัติครอบครัวจึงนาจะตรวจแค stool occult blood 41.) ตอบ 5. ตรวจ HBs Ag ซ้ําอีก 6 เดือน IR Complete heart block ขอนี้มันมีในขอสอบชุดที่ 3 สถาบันจํามา แตไมตรงกันงะ โจทยวา หญิงจะมาสมัคร S พยาบาล ตรวจเลือดพบ HBsAg + และ HBsAb – (นาจะหมายถึง anti-HBs -ve) สวนชุด AJ ของคณะเราเปน anti-HBc IgM –ve LBBB การแปลผล คือ HBsAg + แสดงวา มีการติดเชื้อ HBV IR anti-HBc IgM –ve แสดงวา ไมใช recent infection IR anti- HBs -ve แสดงวา ไมมีภูมิ มาตัด choice กัน S ขอ 1 ให HBV vaccine ผิดแหงๆ เพราะติดเชื้อไปแลว จะมาให vaccine อะไรกัน Premature atrial beat ขอ 2 ให IFN ผิด เพราะจากโจทยบอกวา LFT normal ซึ่งถา ALT ปกติ หรือ 37.) ตอบ 5. C.difficile เพิ่มขึ้นนอยกวา 2 เทาของคาปกติ ก็ให follow up ยังไมตองรักษา การ follow up ยังไงก็ เนื่องจากมีประวัติเขาไดกับ antibiotic associated diarrhea และมีลักษณะเปน mucous ขึ้นกับวาHBeAg +veหรือ–ve [จะให IFN เมื่อ HBV DNA>20000 หรือ ALT > 2X ULN] diarrhea ขอ 3 และ 4 ผิดคูกัน เพราะการตรวจ AFP ,U/S liver เปนการตรวจ HCC surveillance ใน high risk group ซึ่งไดแก ชาย >40 ป , หญิง >50 ป, มี advanced fibrosis/ cirrhosis, มี family Hx of HCC ซึ่งจะตรวจทุก 6 เดือน
  • 9. จึงตอบ ขอ 5 ตรวจ HBs Ag ซ้ําอีก 6 เดือน ถายังมี HBsAg +ve อยู แสดงวาเปน Lubricant eg. Mineral oil หลอลื่น แตใขอเสีย คือ ลดการดูดซึมวิตามินที่ละลายใน HBV carrier เพราะวาสวนใหญรางกายจะสามารถกําจัดเชื้อไปไดเอง ไขมัน เพราะมันจะไหลออกมากับอุจจาระเลย หามใชรวมกับ surfactant laxatives 42.) ตอบ 4. delirium tremens Note stimulants, lubricant มีการใชเพื่อลดนน. ถือเปน laxatives abuse และมีผล เนื่องจากผูปวยมีประวัติดื่มเหลามานาน และหลังเขารักษาในรพ. แพทยไดNPOทันที รบกวน normal bowel habit J จึงเกิดอาการอาละวาดดึงสายน้ําเกลือ Surfactant laxatives eg. castor oil ทําให stool นุมออกงายขึ้น castor oil ใชเตรียม RA อาการDT ไดแก confusion, diarrhea, disorientation and agitation, perceptual bowel เพราะออกฤทธิ์เร็ว (1-3 hr) แตมีขอเสียคือ electrolyte imbalance ได RI disturbance(visual), autonomic hyperactivity (high pulse, blood pressure, and rate of Saline and Osmotics laxatives eg. MOM ; milk of Magnesia, Unison suppository SI breathing) (Na) ออกฤทธิ์เร็วใน 3 hr ทําให osmolarity in colon สูงขึ้นจึงดึงน้ําออกมาคลุกเคลากับ สาเหตุ: ผูปวยมักมีประวัติดื่มalcoholมานาน และมีการหยุดดื่มทันที ทําใหเกิดอาการ อุจจาระ ซึ่งมันก็จะดึงน้ําออกมาพรอมกับ electrolyte ดวย ดังนั้นจึงมีขอเสีย คือ electrolyte AJ withdrawal Delirium tremens มักเกิด3-7 วัน หลังการดื่มครั้งสุดทาย หากไมไดรับการ imbalance และอาจมีการดูดซึม Mg เขากระแสเลือดได เกิด SE ของ Mg, ไมควรใชใน IR รักษาอาจทําใหเสียชีวิตได(mortality rate35%) สําหรับผูปวยที่มีความเสี่ยงสูงไดแก ผูปวยโรคไต extreme fever, fluid and electrolyte imbalance, or intercurrent illness such as occult 44.) ตอบ 1. ENT exam IR trauma, pneumonia, hepatitis, pancreatitis,alcoholic ketoacidosis,Wernicke-korsakoff neck mass ที่ นานกวา 2 wk ควร refer เพื่อตรวจ ENT exam ทุกราย S syndrome 45.) ตอบ 2. Plasmodium vivax AJ การรักษา : ใหยาในกลุมbenzodiazepines P. falciparum P. vivax P. malariae P. ovale 43.) ตอบ 1. bulk forming agent Form ที่พบได - Ring form พบไดทุกระยะ พบไดทุกระยะ พบไดทุกระยะ IR ยาระบาย (laxatives) มีดังนี้ ใน peripheral (Trophozoites) IR Fiber supplements or bulk laxatives eg.Metamucil เปน laxatives ที่ปลอดภัยที่สุด blood smear - Gametocyte S กลไกคือ fiber จะดูดน้ําทําใหอุจาระขยายขนาดใหญขึ้นไปกระตุน reflex ใน colon ดังนั้น ลักษณะของ - normal - ขนาดใหญ สี - ขนาดปกติ - ขนาดปกติ จึงตองกินน้ําตามเยอะๆ ถากินน้ําไมพอก็จะอุดตันลําไสได ขอเสียคือใชเวลาหลายวันกวา RBC size&shape จางลง หรือเล็กกวา หรือใหญ สีจาง จะไดผล และพวก fiber จะลดการดูดซึมของยาและวิตามิน - Maurer’s dots - Schuffner’s ปกติ ลง Stimulants eg. Bisacodil (Dulcolax), Senokot กระตุน n.plexus ในลําไสทําให colon dots - ziemann’s - tear drop RBC m. หดตัว(กระตุน reflex โดยตรง) แตถาใชนานๆ n.plexus จะถูกทําลายทําใหตอมาทองผูก dots - Schuffner’s ไปเลย จึงไมควรสั่งใหคนไขที่ทองผูกเปนประจําใช dots จํานวน 20 (16-32) 16 (12-24) 8 (6-12) 8 (6-12)
  • 10. merozoites 46.) ตอบ 4. artisunate + mefloquine ใน RBC 47.) ตอบ 1. Acyclovir รูปรางของ Crescent form Spherical Spherical Spherical shape ขอนี้หลัก ๆ คือใหดู CSF profile ซึ่งพบวา glucose และ protein ไมตางจากคา Gametocyte shape shape ปกติมากนัก และ wbc ก็มี Lymphocytosis ถึง 95% (มันไมมีคา wbc มาให แตคิดวาคงไม J (มี growing สูงมาก) G/S ก็ไมพบ จึง R/O bacteria ออกไปไดเลย สวน fungus , TB มี lymphocytosis RA trophozoites ได แตควรมีการเปลี่ยนแปลงของระดับ glucose และ protein ที่ชัดเจนกวานี้ จึงคิดถึงนอย RI เปน band (Indian ink ก็ไมเจอ crypto , choice ก็ไมมี TB) นาจะเปน viral มากที่สุดนะจะ SI form) 48.) ตอบ ไมมีรูปอะ เลยไมรูจะเฉลยอะไรแตจะสรุปใหแลวกันนะ Complication cerebral malaria ลักษณะ P.falciparum P.vivax P.malariae P.ovale AJ Treatment infected rbc ปกติ ใหญ ติดสีจาง ปกติหรือเล็ก ใหญ,รี. IR 1. P. falciparum fimbriation - with complication : IR ระยะของเชื้อที่ มักพบเฉพาะ พบทุกระยะ พบทุกระยะ พบทุกระยะ - Quinine dihydrochloride 20 mg/kg IV drip in 4 hr (loading dose) then ตรวจพบ ring form S 10mg/kg IV drip in 2 hr q 8 hr for 7 days Ring form 1/6-1/5 of rbc 1/3 of rbc 1/3 of rbc 1/3 of rbc AJ - Artesunate 2.4 mg/kg IV then 1.2 mg/kg q 12 hr จนครบ 600 mg วงบางมาก วงบาง วงหนา วงคอนขางหนา IR - no complication : - 2 chromatins - Quinine sulfate 600 mg oral q 8 hr + Tetracycline 250 mg oral q 6 hr for 7 IR - multiple days (ในเด็กให quinine sulfate 10 mg/kg oral q 8 hr ใน 4 วันแรก then 15-20 mg/kg infection S q 8 hr for 4 days) - marginal - Artesunate 100 mg oral then 50 mg oral q 12 hr จนครบ 600 mg then form Mefloquine 750 mg oral then 500 mg oral หลัง dose แรก 6 hr Growing เหมือน ring คลายอะมีบา Band form Cytoplasm เปน 2. P. vivax & P. ovale trophozoite form Pseudopod compact form วงหนา, - Chloroquine + Primaquine oral for 14 days (หากเปน G-6-PD จะลด dose มาก pseudopod primaquine) Vacuole ชัด นอย. Vacuole 3. P. malariae - Chloroquine 600 mg (10 mg/kg ในเด็ก) oral
  • 11. ไมชัด สาเหตุ แต the most common cause of erythema nodosum คือ Bacterial infections Schizont 8-32(24) 12-24(16) 6-12(8), 4-12(8) โดยเฉพาะ Streptococcal infections พบทั้งในเด็กและผูใหญ ในผุใหญที่พบบอยอีกอันคือ rosette sarcoidosis เพราะThe most common cutaneous manifestation of sarcoidosis is erythema Gametocyte Crescent กลม กลม กลม nodosumนั่นเอง นอกจากนี้ infectionsในผูใหญ(โดยเฉพาะใน developing countries อยาง J ประเทศเรา) ตองคิดถึง Tuberculosisดวย(จริงๆยังมีอีกหลายเชื้อเลย สามารถคนดูใน RA 49.) ตอบ 1. branching septate hyphae Tinea cruris (ขาหนีบ) การติดเชื้อราในกลุม dermatophytes หรือ กลาก ลักษณะ หนังสือ หรือ emedicineได) RI lesion เปน erythematous active border with central clearing, scaly (ขุยๆ) pruritic (คันๆ) ดังนั้น investigationในรายนี้ คือการหาการติดเชื้อนั่นเอง ซึ่งอาจทําไดหลายอยาง เชน SI Ix คือ KOH พบ branching septate hyphae with arthrospore Rx คือ clotrimazole ทา bid throat culture,stool examination, blood cultures ตามเชื้อที่สงสัย ซึ่งในผูใหญเราสงสัย 2-3 wk (Tinea ใชยาทาก็พอ ยกเวน Tinea capitis (หนังหัว) และ Tinea unguim (เล็บ) ตอง Tuberculosis กับ sarcoidosis ดังนั้นตองสง film chest x-ray ดวย จึงตอบขอ 1 AJ ใหยากินดวย เชน griseofulvin, itraconazole) 51.) ตอบ 4. Praziquantel (จากภาพเปนไขของ Opisthorchis viverrini) IR Pseudohyphae with budding yeast = Candida 52.) ตอบ 2. Euglobulin test Bleeding time I/C : ผูปวยที่มีเลือดออกผิดปกติ ที่นาจะเกิดจาก primary IR Short septate hyphae with oval yeast (spaghetti and meatball) = Malassezia furfur (โรคเกลื้อน Pityriasis versicolor) hemostatic defect แตมีจํานวนเกร็ดเลือดปกติ S 50.) ตอบ 1. CXR patient preparation : AJ จากโจทย นึกถึง Erythema nodosum (EN) มากที่สุดเนื่องจากอาการแสดงของผูปวย - แขนของคนไขขางที่จะตรวจ ตองไมไดอยูระหวางการได IV fluid ใดๆ ทั้งสิ้น IR เขาไดdefinitionของโรคนี้เลย คือ - คนไขที่รับประทานยา Aspirin ตองหยุดยา 7 วันขึ้นไป กอนรับการตรวจ Erythema nodosum (EN) is an acute, nodular, erythematous eruption that usually - ไมตรวจในผูปวยที่มี Hct ต่ํากวา 30 % IR is limited to the extensor aspects of the lower legs. วิธีการวิเคราะห (methodology) : Modified Ivy method โดยควบคุมความดันเลือด S โดยมีclinical presentation คือ The eruptive phase of erythema nodosum begins ของแขน ผูปวยดวย เครื่องวัดความดันโลหิต ที่ 40 มิลลิเมตรปรอท และใชมีดปลายแหลม with flulike symptoms of fever and generalized aching ก็คือมี Febrile illness with เจาะผิวหนังใหเปนแผลลึก 3 มิลลิเมตร กวาง 2 มิลลิเมตร จับเวลาตั้งแตเลือดเริ่มไหลจน dermatologic findings includes abrupt onset of illness with initial fever, followed by เลือดหยุดไหล โดยใช กระดาษกรองซับเลือด a painful rash within 1-2 days. ก็เหมือนในโจทยนั่นเอง ----> ผูปวย Dengue มี thrombocytopenia อยูแลว โดยโรคนี้เปน delayed hypersensitivity reaction and may occur in association with Euglobulin lysis time I/C : ผูปวยที่สงสัยภาวะ increased fibrinolysis several systemic diseases or drug therapies, or it may be idiopathic. โดยมีสาเหตุไดหลาย ----> Hemostasis depends on the balance between coagulation and fibrinolysis Laboratory tests for assessment of the coagulation system
  • 12. - Coagulation Screening tests , Activated partial thromboplastin Time , เชื้อ Naegleria fowleri เขาสูรายกายทางโพรงจมูก มักจะเกิดเมื่อมีอากรสําลักน้ํา และ Prothrombin time , Thrombin time , Fibrinogen , Fibrinolysis , Screening tests , เชื้อจะขึ้นสมองโดยผานทางเสนประสาทรับกลิ่น (olfactory nerve) ทําใหเกิดเยื้อหุมสมอง Euglobulin clot-lysis time , Dilute-whole-blood clot-lysis time อักเสบ (Primary amoebic meningo-encephalitis หรือ PAM) N. fowleri มีรูปรางลักษณะ 2 ผลแทรกซอนของโรค Dengue ระยะ ไดแกระยะ trophozoite ซึ่งอาจมีรูปรางแบบอะมีบา (ameboid form) กับแบบมี J 1. Prolonged shock หนวดสําหรับวายน้ํา (flagellate form) และระยะ cyst เชื้อเจริญเติบโตไดดีที่อุณหภูมิสูง RA 2. Fluid overload ระยะฟกตัวของโรค 3-7 วัน ผูปวยจะมีอาการปวดศีรษะมาก เจ็บคอ คลื่นไส อาเจียน RI 3. Massive bleeding เพอ ไขสูง คอแข็ง งวงซึม อาการจะทรุดอยางรวดเร็ว และมักจะเสียชีวิตภายใน 10 วัน SI 4. Encephalopathy จาก hepatic encephalopathy โรคนี้สวนใหญพบในคนวัยหนุมสาวทีมีสุขภาพ 53.) ตอบ 1. Melioidosis 56.) ตอบ 2. Chikungunya AJ เนื่องจาก fever 2wk, FBS 260 (นาจะมี U/D เปน DM), BUN/Cr สูง (อาจมี renal ติดตอโดยยุงลาย Aedes aegypti อาการไดแก ไขสูงเฉียบพลัน ผื่นแดงตามรางกาย อาจ IR impairment อยูเดิม), ตับโต มามโต ; u/s พบ hypoechoic ก็นาจะบงถึง abscess ซึ่งมี คันรวมดวย conjunctival injection,migratory polyarthritis โดยมักเปนที่ขอเล็กๆ เชน multiple ที่มาม ; จากประวัติทั้งหมดนี้เปน typical natural Hx of “ Melioidosis ” ขอมือ ขอเทา อาการหายภายใน 1-2 wk ไมรุนแรงถึงshock อาจพบpetechiae หรือ IR Hx : ผูปวยมี U/D เชน DM, renal fail, thalassemia อาชีพที่สัมผัสดิน เชนทํานา ตรวจ tourniquet +ได ขอแตกตางจาก Dengue คือ ไขสูงเฉียบพลันกวา และระยะไขเพียง 2 วัน, S พบบาดแผลที่ขา/เทา มาพบแพทยโดยไขสูง จํานวนpetechiaeใน tourniquet testนอยกวา,ไมพบconvalescent rash,พบMP AJ Ix : พบ abscess ตาม organ ตางๆ ที่พบบอย : liver, spleen, skin rash,conjunctival infection,myalgia,arthalgiaไดบอยกวา,ชักจากไข15%ซึ่งมากกวา Rx :1. Acute : Ceftazidime iv or Imipenem or Meropenem นาน 2 wks dengue3เทา ระบาดชวงฤดูฝน เปนไดทุกกลุมอายุ รักษาโดย supportive treatment IR 2. ตอมา : Co-trimoxazole + Doxycline (ในเด็กให Augmentin แทน) นานรวม Rickettsial infection แบงเปน3กลุมไดแก spotted fever,typhus,scrub typhus อาการ IR ระยะ acute = 20 wks เดนคือไขสูงหลายวัน อาการอื่นๆ เชน ปวดหัว ปวดกลามเนื้อ ออกผื่น คลื่นไส อาจมี S 54.) ตอบ 3. Strongyloides stercolaris eschar diag โดย serology รักษาโดย doxycycline ผูปวยที่มีการติดเชื้อ stongyloid อยูกอน เมื่อไดยา steroids ซึ่งกดภูมิคุมกันทําใหเกิด Rheumatic heart diseaseลิ้นหัวใจถูกทําลายจากกระบวนการเกิดโรคซึ่งเริ่มตนจาก disseminated strongyloidiasis คือแพรกระจายไปหลาย system ผูปวยอาจมาดวย การเจ็บคอจากการติดเชื้อ streptococcus เปนการดําเนินโรคหนึ่งของ RF acuteมักมาดวย abdominal pain, distension, shock, pulmonary and neurologic complications, septicemia carditis(ไมคอยมีผลตอ hemodynamic) chronicมักมาดวยmitral stenosis ดังนั้นจึงควรตรวจอุจจาระหา larva ของมันกอนเริ่มยาเสมอ 55.) ตอบ 1. N. fowleri
  • 13. 57.) ตอบ 2. xertic eczema ‐ ถาไมเสี่ยง ให NG หรือ PEG โดยถาตองใหนานเกิน 1 เดือน พิจารณาใหทาง อาจเรียก asteatotic eczema หรือคนทั่วไปเรียก winter itch พบมากในผูสูงอายุ PEG โดยเฉพาะชวงอากาศหนาวหรือแหงกวาปกติ สามารถพบบริเวณหนาแขงไดบอย เนื่องจากรายนี้ ทางเดินอาหารใชได แตกินอาหารเองไดนอย มีความเสี่ยงต่ําในการ เนื่องจากเปนบริเวณที่มี subcutaneous tissue นอยกวาสวนอื่นๆ ของรางกาย aspirate ดังนั้นสามารถใหผาน NG / PEGได แตเนื่องจากรายนี้นาจะตองใหอาหารทางนี้ J 58.) ตอบ ? เกิน 1 เดือนดังนั้น นาจะใหทาง PEG RA Zinc deficiency: Abnormal Growth, Hypogonadism, Skin Problems, Weight Loss, 61.) ตอบ 5. ควบคุมอาหาร + ลดน้ําหนัก RI Diarrhea, Emotional Instability, Hair Loss เนื่องจาก primary hypertension ควรใหการรักษาดวย life-style modifications (weight SI Retinol deficiency: Bitot spots, nyctalopia, Dry skin, Dry hair, Pruritus, Broken loss, smoking cessation, salt reduction) กอนหากยังไมสามารถควบคุมระดับความดัน fingernails, Keratomalacia, Xerophthalmia, Follicular hyperkeratosis โลหิตไดตามเปาหมายจึงจะพิจารณาใหยา anti-hypertensive drugs ตอไป AJ Selenium deficiency:keshan disease, Kashin-Beck disease,(myocardial necrosis, Pediatrics IR atrophy, degeneration and necrosis of cartilage tissue), hypothyroidism 1.) ตอบ 2. พาไปพบจิตแพทย 59.) ตอบ 1. thiamine IR คิดถึง ADHD (Attention deficit hyperactive disorder) มี 3 กลุมอาการหลัก กอน Wet beriberi (cardiac beriberi) โดยพบในผูที่ไดรับวิตามิน บี1 นอยกวา 0.2 อายุ 7 ป ( แตเด็กคนนี้ 8 ป อานะ) คือ 1. ซน (Hyperactive) , 2. ขาดสมาธิ (Inattentivit) , S มก./1,000 กิโลแคลอรี่ มีอาการสําคัญคือบวม ระยะแรกจะรูสึกขาหนัก เดินลําบาก บวม 3. หุนหันพลันแลน (Impulsivity) AJ เล็กนอยที่ขาและหนา เนื่องจากมี lactic acid และ pyruvic acid คั่ง เพราะ oxidative Rx. Medication (stimulant;Methylphenidate) + Behavior modification IR decarboxylation นอยลง ทําใหเซลลอักเสบและมีสารน้ํารั่วออกมา และหัวใจขาดพลังงาน Impact . การเรียน สังคม การปรับตัว self esteem เพราะใชสารทั้งสองไดลดลง IR 2.) ตอบ 3. stranger anxiety 60.) ตอบ 4. PEG (Percutaneous endoscopic gastrostomy) เพราะเด็กอายุ 9 mo นาจะมี stranger anxiety เพราะฉะนั้นมักจะกลัวคนแปลก S เรื่อง nutrition ตองประเมินกอนวา GI function ไดหรือไม ถาได ก็ดูวากินเองไดมาก หนา ไมไดยอมใหทุกคนอุม จึงคิดวาขอนี้เปนพัฒนาการที่ผิดปกติสําหรับเด็ก 9 mo นอยแคไหน เด็กอายุ 9 mo : Gross&fine motor -> pull to stand,peek- a- boo (จะเอ), mature - กินไดเองมากกวา 75% ใหกินเอง pincer grasp, นั่งไดมั่นคง, คลาน / Language-> immature jagoning / Social-> จะเอ, - กินได 50-75% ใหกินเองและ oral supplement object permanent / Stranger anxiety เริ่มตอนอายุ 6-9 mo ถึง 2 yr เดนตอนอายุ 9 mo - กินไดเองนอยกวา 50% ให enteral feeding 3.) ตอบ 4. ลดนมเหลือวันละ 2 มื้อ ซึ่ง enteral feeding ตองพิจารณาวามีความเสี่ยงตอ aspiration ไหม เด็ก 2-6 ขวบ คิดน้ําหนักโดย 2x+8 คนนี้ 2ขวบ ได 12 Kg น้ําหนักเกินนะ เพราะหนัก ‐ ถาเสี่ยง ให ND/ NJ tube
  • 14. 16 Kg สวนสูง ใช 6x+77 กอเกือบพอดีเลย -> คิดวาขอนี้นาจะตอบไปในทางลด จะลดมือ ที่ไมตองใชทักษะฝมือ หรือกึ่งใชฝมือ แตอาจตองการคําแนะนํา และการชวยเหลือบางเมื่อ อาหารก็ไมได เพราะเด็กจะไดรับสารอาหารไมครบ แตถาลด นมก็พอได ประสบความเครียด 4.) ตอบ 3. รับ OPV, DTP IQ 35-49 moderate MR สามรถเรียนจบ ป.1-ป.2 สามารถฝกอบรมได (trainable) ใน ทักษะการชวยเหลือ ดูแลตนเอง เรียนรูที่จะเดินทางไดดวยตนเองในสถานที่ที่คุนเคย และ J Birth 1mo 2mo 4mo 6mo 9mo 12mo 18mo 4-6yr >4- ฝกอาชีพไดบาง สามารถทํางานที่ไมตองใชทักษะฝมือ แตควรอยูภายใตการกํากับดูแล RA 6yr อยางใกลชิด RI BCG IQ20-34พอจะฝกฝนทักษะการดูแล ตนเองเบื้องตนไดบางแตนอย ดํารงชีวิตอยูใน SI HBV1 HBV2 HBV3 สังคมภายใตการควบคุมดูแลอยางเต็มที่ การทํางานตองการโปรแกรมในชุมชน หรือการ ใหความชวยเหลือที่พิเศษเปนการเฉพาะ DTP1& DTP2& DTP3& DTP4& DTP5& AJ IQ <20 พัฒนาการลาชาอยางชัดเจนในทุกๆดาน มักมีพัฒนาการดานการเคลื่อนไหว OPV1 OPV2 OPV3 OPV4 OPV5 IR และฝกการชวยเหลือตนเองไดบาง มีขีดจํากัดในการเขาใจและการใชภาษาอยางมาก MMR1 MMR2 ตองการความชวยเหลือ ดูแลอยางใกลชิดตลอดเวลา IR JE1&2 7.) ตอบ 3. Amblyopia S หางกัน จากโจทยผูปวยมีภาวะ misalignment ของ rt eye (internal strabismus) complication ที่ AJ 1-4 อาจเกิดขึ้นไดคือ เนื่องจากวาเด็กนั้นนารัก มีการปรับตัวอยางดีเยี่ยม พอตา 2 ขางมองได ไมเหมือนกัน จากอะไรก็ตาม... สมองก็เลยมีการปรับใหตาที่มีปญหานั้นลดการมองเห็นลง IR สัปดาห DT ไป เลยกลายเปนตาขี้เกียจ amblyopia นั่นเอง IR คําอธิบาย Expanded program of immunization (EPI) Key อยูที่วา เราตอง detect กอนอายุ 7-8 ป แลวรีบรักษาซะ เพราะถาเปนแรวจะ รักษา S 5.) ตอบ 1. TORCH titer ไมได เพราะ เด็กมี increase muscle tone , hyperreflexia , delay development (6เดือน ยังชัน Amblyopia หมายถึงภาวะที่ตามี visual acuity ลดลงโดยไมมี organic lesion คอไมได) head circumference มากผิดปกติ ทําใหนึกถึง involve CNS จึงคิดถึง TORCH ถาตาขางหนึ่งมี VA ที่แกไขแลว (ดวย pinhole หรือแวน) นอยกวาตาอีกขางหนึ่ง เทากับ infection 2 แถวของ Snellen Chart ขึ้นไป 6.) ตอบ 2.ออกใบรับรองเขาเรียนไดโดยมีอาจารยพิเศษ 8.) ตอบ 1. Viral croup IQ 50-69 (60) เปน mild MR สามารถเรียนรูได (educable) สามารถเรียนจนจบชั้น Viral croup (laryngotracheobronchitis) common ในเด็กอายุ 6 เดือน – 4 ป ประถมปลายได สามารถฝกทักษะดานสังคมและอาชีพ พอที่จะเลี้ยงตัวเองได เปนแรงงาน
  • 15. เปน Inflammation and edema of the subglottic area causing airway obstruction in the 11.) ตอบ 3. CXR PA, lateral larynx, trachea, or bronchi. ตรวจรางกายมี inspiratory stridor มี sign ของ upper airway เด็กมี inspiratory stridor ซึ่งเปนลักษณะของ upper airway obstruction และมี rhonchi obstruction ; สาเหตุสวนใหญเกิดจากเชื้อไวรัส(Parainfluenza virus) แตก็อาจเกิดจาก ซึ่งเปนลักษณะของ lower respiratory tract obstruction โรคที่ทําใหเกิดลักษณะเชนนิ้ คือ bacterial infection ได เชน S. aureus, Mycoplasma pneumoniae, S. pneumoniae, S. Croup = Viral laryngotracheobronchitis (larynx = upper, bronchus = lower) การตรวจ X- J pyogenes, Hib (bacterial croup จะ present ดวย high fever & more severe resp. distress) ray ที่ neck AP พบ pencil sign, steeple sign Lateral พบ ballooning of hypopharynx จึง RA acute epiglottitis มักเกิดในเด็กอายุ 2-7 ป ทําใหเกิด supraglottic stenosis มี sign ของ ตอบ CXR PA,lateral เพราะสามารถดูสวนคอไดดวย RI upper airway obstruction ไดเชนกัน และลักษณะ stridor เปน inspiratory stridor ผูปวยจะ 12.) ตอบ 3. SVC obstruction SI มีไขสูง, muffle voice + 4Ds symptoms (drooling, dyspnea, dysphagia, dysphonia) classic symptoms and signs become more obvious. Dyspnea is the most common Whooping cough (pertussis) เปนการอักเสบของทางเดินหายใจสวน bronchus มัก symptom , Other symptoms include facial swelling, head fullness, cough, arm swelling, AJ เปน expiratory stridor มีอาการไอมาก อาจตรวจพบ subconjuctival hemorrhage, rectal chest pain, dysphagia, orthopnea, distorted vision, hoarseness, stridor, headache, nasal IR prolapsed ได lung มี croase crepitation, ไมคอยตรวจพบวามี tachypnea การตรวจ CXR stuffiness, nausea, pleural effusions, and light-headedness. อาจพบ peribronchial thickening findings : venous distension of the neck and chest wall, facial edema, upper IR 9.) ตอบ 3. Retropharyngeal abscess extremity edema, mental changes, plethora, cyanosis, papilledema, stupor, and even S ตรวจรางกายพบ Bulging posterior pharynx coma. AJ 10.) ตอบ 2. ICD 13.) ตอบ 2. Acute Renal Failure ผูปวยนาจะมี Pneumonia with parapneumonic effusion เยอะพอสมควร trachea ถึง จากโจทย นาจะเปน MAHA ที่พบบอยในเด็กคือ HUS โดยจะมีอาการนําคือ มีไข IR จะshift ไมแนใจนาจะ 2. ICD นะ เพราะถา 1. mycoplasma titer จาก mycoplasma ถายปนมูกเลือด โรคนี้เปน vascular-occlusive disorder จึงมีการแตกของเม็ดเลือดแดง IR pneumomiae จะเปนแบบ walking pneumonia คืออาการไมคอยมาก แต CXR ดู severe อะ เกล็ดเลือดต่ําได & จาก U/A แสดงวานาจะมี intravascular hemolysis ก็ควรระวัง Acute S ก็พบวาทําใหเกิด pleural effusionได แต rare อะ อาการของ mycoplasma จะ non-specific Renal Failure onset - insidious, with fever, malaise, headache, and cough. Cough is a hallmark of M 14.) ตอบ 4. ANA pneumoniae infection และก็มี extrapulmonary manifestation ได เชน มีผื่น maculo เหมือนมี multiorgan involvement ( Hematology, mucosal involvement , kidney papular, erythema multiforme, bullous myringitis, musculoskeletal เชน ปวดขอ etc. [ involvement) เหลืออีกหนึ่งอยางจะครบ criteria SLE แลว ก็ตรวจANA ไปเลยดิจะได nasopharyngeal swab ไมรูนะวาขอนี้เคาตั้งใจจะใหเปน Flu เพราะชวงนั้น Fluระบาด] วินิจฉัย